Msk mcq
Msk mcq
Msk mcq
Q4. A 25-year-old male presents with knee and wrist joint pain (and effuison?) with no other symptoms and normal
cardiovascular system , culture were taken and showed negative diplococci .
a. Staphylococcus
b. Streptococcus
c. Neisseria gonorrhoeae
Q5. 17-year-old girl presents with arthralgia, photosensitivity, malar rash and proteinuria
a. Vasculitis 2ary to RA
b. lupus nephritis
c. UTI
d. Psoriatic arthritis
Q8. A 32-year-old man known case of hypertension on calcium channelblocker presents to your clinic complaining of 2
days duration of only right ankle pain and swelling. The pain started suddenly and became severe within few hours with
inability to walk and bear weight. He denies history of preceding trauma, fall or recent infection. He had similar attack 2
years ago in the left ankle that resolved gradually over 1week. His physical exam showed swelling and redness of the
ankle joint with limited range of motion passively and actively.
Q10. A 31-year-old man is evaluated in the emergency department for fever and red painful eyes. He reports a 1-month
history of intermittent painful oral and genital ulcers, knee pain, and fever and 1 week of bilateral eye redness, pain and
photophobia. On physical examination, temperature is 38.3 and pulse rate is 100/min. Slit lamp examination reveals
white cells in the anterior chambers of both eyes. There are aphthous ulcers on the tongue and one aphthous ulcer on
the scrotum. Swelling and warmth are noted in both knees. The heart, lung, abdominal, and neurologic examinations are
normal. Which of the following is the most likely diagnosis?
a. Behcet syndrome
b. Crohn's disease
c. Sarcoidosis
d. Systemic lupus erythematosus
e. Spondyloarthritis
- Lecture: Rheumatology
_______________________________________________________________________________________________
Q13. اﻟﺳؤال ﻛﺎن ﻣﯾس طوﯾﻠﺔ و ﯾﺳﺄل ﻋنsle, ﻓﻲ اﻟﻧﮭﺎﯾﺔ ﻛﺗب اﯾش اﻻﻧﺗﻲ ﺑدي اﻟﻠﻲmost specific ﻟلsle
- Lecture: Rheumatology
_______________________________________________________________________________________________
Q15. Patient presents with joint pain tenderness swelling erythema fever and Limitation ROM
CBC: leukocytosis
a. Gout
b. Pseudogout
c. Ankylosing spondylitis
d. Septic arthritis
- Correct answer: Septic arthritis
- Lecture: Rheumatology
- Explanation: Typical patient presentation, confirmed with leukocytosis and extremely high cell count
_______________________________________________________________________________________________
Q16. Patient presents to the ER after car accident, he was drunk and got into an accident going 140 km/hour.
Which of the following is appropriate in regard to removing the clearing the C spine?
Q20. 40-year-old man IV drug users complaining of mid low back pain for 3 months,........... He loss unintentionally 10 kg
last 2 weeks. What is the most likely diagnosis?
Q26. A 65-year-old lady C/O few years knee pain and there is varus deformity and tenderness at medial and lateral Joint
line. Which of the following is the most likely diagnosis?
Q27. 55-year-old had difficulty in raising his hand above his head, what muscle are affected?
a. Supraspinatus
b. Infraspinatus
c. Teres minor
d. Deltoids
Q29. 30 years old woman involved in a motor vehicle accident, she sustained tibia fracture with 2CM skin laceration on
the site of fracture, the wound is clean no contamination, what is the best treatment option?
Q30. Patient after hip replacement surgery after 24 hours with shortness of breath. what is the most likely diagnosis?
a. Pulmonary embolism
b. Atelectasis
4. A 65-year-old man was admitted 3 days ago to coronary care unit with
acute MI and was treated with thrombolytic therapy and was started on
low dose Aspirin, metoprolol and valsartan. This morning, he
developed acute and sudden left knee pain and swelling. His cardiac
status is control. His vital signs are stable. Left knee exam showed
moderate to severe effusion with limited range of motion and
tenderness of the knee joint line. The rest of the MSK exam was
unremarkable.
What is the best investigation modality to reach your final diagnosis?
A. CBC and Blood culture
B. Serum ANA, RF and anti-CCP antibodies
C. Knee joint aspiration and synovial fluid analysis
D. Knee X-ray
12. A 25-year-old man is evaluated for the gradual onset of bilateral low
back pain without radiation to the lower extremities daily, with
increasing severity over the past year. The pain now awakens him
during the night 2 to 3 times per week, with morning stiffness lasting
more than an hour. His pain improves with exercise. He takes ibuprofen
which provides mild relief.
On physical examination, vital signs are normal. Limited lateral bending
bilaterally and a reduction in forward flexion at the lumbar spine are
noted. The remainder of the examination is normal. An antero-posterior
plain radiograph of the pelvis and sacroiliac joints is unremarkable.
Which of the following is the most appropriate diagnostic test to
perform next t?
A. Bone scan
B. CT of the lumbar spine
C. MRI of the sacroiliac joints
D. Radiography of the hip joints
E. All the above
B
15. Young female typical presentations of SLE, what test to screen?
A. ANA
B. ACPA
C. ANCA
D. Anti RNP
E. Anti-Cardiolipin
20. Patient complaining of back pain for 2-days, he feels tightness and he
can’t bend his back forward, he scale the pain 7 out of 10
A. Straight leg raise test
B. Compression test
C. Schober test
D. Slump test
21. Patient with pain in his PIP and MCP with bilateral swollen wrist for 4
weeks he has mild pain in the right upper quadrant with elevated liver
enzymes, what is the most likely diagnosis?
A. Autoimmune hepatitis
B. Hepatitis B induced arthritis
C. Primary sclerosing cholangitis
22. A 28-year-old female patient diagnosed with SLE 2 years ago. She is on
hydroxychloroquine 200 twice a day, aspirin 81 once daily and
prednisolone once daily. She is doing well but in remission.
Which of the following statements is true regarding this patient?
A. Discontinue all medication
B. She can’t be pregnant, she is infertile
C. She shouldn't due to fetal and maternal death
D. Patient’s disease should be controlled and in remission for six months
before pregnancy
E. She can continue all medication during pregnancy
D
23. A 22y old male complaining of hemoptysis and dyspnea, 5h prior
presentation from brother history, patient have painful uveitis improve
with topical medication, on-and-of gentle ulcer, with acne that is
resistance to treatment. RR: 33, O sat: 87%. What is most common
vasculitis cause pulmonary aneurysm?
A. Takayasu arteritis
B. Behcet disease
C. Giant cell arthritis
D. Granulomatosis polyangiitis
E. Cogner syndrome
24. Female worker (teacher) complaining of back pain and tenderness in the
Sacro-Iliac joint that improves with activity associated with morning
stiffness, she can’t bend forward
What is your initial diagnosis?
A. Lumbar disc prolapse
B. Ankylosing spondylitis
C. Reiter’s syndrome
D. Thoracic disc herniation
E. Para-vertebral muscle spasm
25. A 34-year-old female known case of SLE pregnant for 7 weeks, which of
the following is true:
A. Cyclophosphamide should be avoided due to fetal congenital
malformations
B. Steroids should be stopped
C. Methotrexate can be continued during pregnancy
D. Steroids teratogenicity is more than methotrexate
E. Hydroxychloroquine should be stopped because it’s teratogenic
28. A 15-year-old boy presents to the emergency department after falling off
his skateboard. On physical examination he is unable to dorsiflex or
evert at the ankle. In addition, the patient reports pain and numbness in
the lateral leg and dorsum of the foot. When asked to walk, he raises his
affected leg high off the ground and his foot slaps the ground when
walking. He is diagnosed with a fracture. Which of the following
structures is most likely to be compromised by this fracture?
A. Obturator nerve
B. Common perineal nerve
C. L4 nerve root
D. Femoral nerve
E. Tibial nerve
B
29. An 18-year-old man is injured during a soccer game when the goalie
dives for the ball, but tackles the player on the lateral aspect of his leg.
He is helped off the field and brought to the emergency department,
where he tells, the physician that he heard a pop when he was tackled.
Physical examination reveals an anterior drawer sign and MRI reveals a
torn anterior cruciate ligament and medial collateral ligament.
Which one of the following structures is also likely to be injured?
A. Medial meniscus
B. Lateral meniscus
C. Posterior cruciate ligament
D. Lateral collateral ligament
E. Patellar ligament
30. A 10-year-old girl has pain in her left forearm and fever, patient had
URTI one week ago, she use hydroxyurea. Multiple abdominal pain
episodes, blood culture was taken. Which of the following is most likely
organism
A. E.coli
B. Streptococcus aeruginosa
C. Salmonella
D. Streptococcus pyogens
E. Rhino virus
31. A 49-year-old man is lifting a heavy box above his head when he
experiences a sudden tearing sensation and pain that travels along his
left arm. When he attempts to move his arm, he is not able to medially
rotate it. Which of the following muscles is most likely injured?
A. Infraspinatus muscle
B. Subscapularis muscle
C. Teres minor muscle
D. Supraspinatus muscle
E. Deltoid muscle
B
32. 12 hours after ORIF for forearm fracture in a 20-year-old patient which
was uneventful, you were called to evaluate the patient who is having
severe pain that is out of proportion to the surgical procedure, not
relieved by narcotics. The volar compartment is hard and painful to
palpate. Your evaluation highly suggestive of volar compartment
syndrome. Which of the following is true regarding the patient’s
condition:
A. Hypotension is a risk factor
B. Intra-compartment pressure over 40 mmHg is the cutoff for the
diagnosis
C. The patient expected to have pain with passive finger flexion
D. Coagulation disorders is not a risk factor
35. If a patent was able to walk on his toe normally (toe walking) without
difficulty, it indicate which lumbar nerve root was tested with full
power?
A. L3
B. L4
C. L5
D. S1
E. L1
36. A 40-year-old women complaining of (....) lower back pain for 1 week
she is neurologically intact (everything normal) MSK exam also normal,
the patient known case of osteoarthritis..
A. NSAID and close follow up
B. Referral to spinal clinic
C. MRI of the spin
D. Bone scan
39. A 40-year-old male IV drug users present with sever lower back pain,
loss of appetite, weight loss 20 kg in the last month, what is the most
likely diagnosis?
A. Vertebral osteosarcoma
B. Disc infection (Discitis)
40. A 35-year-old male was involved in a motor vehicle crash (MVC) and
sustained fracture dislocation of C6-C7. On examination, you will expect
all the following except:
A. Weak elbow flexion
B. Weak fingers flexion
C. Absent sensation over the ulnar side of the hand
D. Absent triceps reflex
E. Knee extension
41. In trauma patients, what is the best method to protect the thoracolumbar
spine while waiting for radiological investigation?
A. Bed rest
B. Spine board
C. Log rolling
D. Brace
42. A 56-year-old woman is seen in the primary care center with complaints
of mild numbness and tingling in her right hand for the past three
months, most notably in the index and middle fingers. The numbness
and tingling occur when she gets up from bed early in the morning or
after using her hand for prolonged periods. She takes medication for
osteoporosis but is otherwise healthy. Her history is unremarkable
except for a wrist fracture after a fall onto her right hand 2 years ago that
was fixed with open reduction and internal fixation. Her postoperative
course was uncomplicated. Physical examination in the clinic reveals a
well-healed surgical incision on the volar aspect of her wrist, normal
grip strength, and no sensory deficits. X-rays reveals a plate and screws
in adequate alignment, with evidence of bony union at a previous distal
radius fracture site. What is the next best step in her management?
A. Application of a cast immobilization and follow-up x-ray to rule out
occult refracture
B. MRI of the right wrist
C. Surgical release of transverse carpal ligament
D. NSAIDs administration, wrist splint with close follow up
43. You were on call at Al-Noor Hospital and were called to see a 30-year
old male patient who was a driver involved in road traffic accident. He
was driving at 140 Km/hr and lost control of the car because he was
under the influence of alcohol. The car rolled over 8 times and the
patient was ejected from the car and found 10 meters from his car. When
he was brought to the emergency department with a cervical collar and
his Glasgow Coma Scale is 11, the trauma team asked you to clear his
cervical spine. What is your best action?
A. ASK the patient and if he does not have neck pain, remove the
cervical collar
B. Examine the patient and if he is neurologically intact, remove the
cervical collar
C. Do AP, Lateral & Open mouth views of the cervical spine and if there
were normal, remove the cervical spine
D. You can’t remove the cervical spine now because the patient is
alcohol intoxicated
44. A 20-year-old female playing football, she had direct injury to her knee
from another player, she couldn't continue playing, her knee was
swollen, patient CAN NOT extent the knee fully, you are suspecting
either ACL injury or quadriceps. what is the main function of the
Quadriceps muscle?
A. Knee active flexion
B. Knee varus stabilizer
C. Knee active extension
D. Knee valgus stabilizer
45. A 30-year-old male patient was diagnosed with a central disc prolapse at
L4-L4 2 weeks ago. he was treated with pain medication and
physiotherapy. His right leg pain and numbness symptoms were
gradually improving. Three hours ago, he developed bilateral severe leg
pain and weakness and inability to pass urine. He thinks that this is
because of the severity of pain. When you examined him, he has
bilateral leg weakness. The reflexes are absent at both knees. Digital
rectal exam revealed loss of sensation of the perianal area and weakness
of the anal sphincter. What is the most likely diagnosis?
A. Recurrence of the symptoms of his L4-L5 disc prolapse
B. Conus medullaris syndrome
C. Cauda equina syndrome
D. New disc prolapse at another level above L4-L5
C
47. During your call duties at hospital, the nurse called you to see patient,
surgery was performed hip fixation in a 70-year-old male about 4 hours
ago, he was having difficulty breathing, oxygen saturation was 90% in
room air, vital signs is stable, what is your differential diagnosis?
A. Acute atelectasis
B. Pulmonary edema secondary to fluid overload
C. Post-operative pneumonia
D. Spinal shock because patient had epidural anesthesia
septic shock due to surgical site infection
48. The best treatment choice of Distal radius fracture in 65 years old male,
minimal displacement ray is:
A. Open reduction and k-wire fixation
B. Close reduction and above elbow cast application
C. Close reduction and k-wire fixation
D. Application of Mini wrist external fixator
E. NSAID & physiotherapy
49. Which of the following is not risk factor for developmental dysplasia of
the hip?
A. Breach
B. Family history
C. Female
D. Lipoma
50. A patient with valgus knee was seen in orthopedic OPD. Which knee
ligament will be stretched?
A. Medial collateral ligament
B. Medial meniscus
C. Lateral collateral ligament
D. Lateral meniscus
A
Comprehensive 38 (F+M)
Rheumatology
1) A 55-year-old presented with 3 week of left knee pain, no effusion, no history of
trauma, tenderness over the antero-medial just distal to knee joint, which of the
following is the best treatment?
A- Injection left knee with steroid and local anesthesia
B- Injection of aserine bursa with steroid and local anesthesia
C- IV diclofenac for 3 days
D- Prescribe oral steroid for 5 days
METHOTREXATE
4) A 31-year-old woman presents with complaints of fatigue and pain in her both
hands and feet that interferes with her daily activities for the last 3 months, on and
off evening rise of temperature (37.8 C-38 C). She is taking paracetamol tablets which
partially improve her pain and fever. She also noticed skin rash over sun exposed
areas and on her face. On further history taking: she mentions that one month ago
she had a right sided pain that worsens with coughing and sneezing, and improved
by NSAIDs. Findings on examination: a red rash over both cheeks and bridge of the
nose, synovitis of MCPs and PIPs of both hands, MTs of both signs are normal as
well as normal systemic examination. Her CBC showed: WBC=3.8 (N 4-11) Hb: 12.5
(N= 12-15), Platelets= 184 (N= 150-400). Which one of the following treatment
options will you start for patient based on her current disease status?
A- Azathioprine, NSAIDs, small dose aspirin
B- Small dose prednisolone, hydroxychloroquine and continue NSAIDs
C- Methotrexate, hydroxychloroquine, prednisolone
D- NSAIDs with high dose prednisolone
E- Short course of oral and topical steroids
5) A 29-year-old man: 10-week: joint pain. His mother has severe RA. O/E: 20
swollen joints and 30 tender joints. Lab: mild normochromic anemia (hemoglobin
12.1 g/dL), elevated ESR 39 mm/h, negative RF test, weakly positive ANA test (1.2
units, reference range <1.0 units), and increased ACPA (anti-CCP) (>100 units,
reference range <5 units). Which of the following statements is true?
A- You are referring him to a hematologist because of anemia
B- Patients with RA do not have positive ANA tests
C- The ACPA results suggest that he is at higher risk of radiographic progression
than RA patients who have negative ACPA results
D- Since his rheumatoid factor test is negative, it is unlikely that he has RA
E- ACPA antibodies occur only in RA
6) A 72-year-old Male came to ER after fall, complaining of left hip pain, can’t stand
and pain at rest. He has recently diagnosed with lymphoma which he take
chemotherapy. Seven years ago, he did hip replacement for osteoarthritis, he has
venous central line on chest. on PE the hip is warmth, tender and he has a fever and
ESR= 73 and Uric acids 8.2 Dx?
A- Gout flare
B- Hemarthrosis
C- Hip dislocation
D- Prostatic joint infection
9) A 55-year-old presented to the ER 7 days ago with gout flare in his left 1 MTP, and
received all management in the ER, he was diagnosed with gout 10 years ago, since
then he had left ankle attack, right knee, right first MTP, Last flare 1 year ago, on
examination, there is no tophi , no erythema, only mild tenderness, otherwise he is
doing well. Lab investigation: BUN 40 (N less than 20), Cr 1.1 (N 0.9-1.2) uric acid 6.8
(N less than 6). Which of the following is most appropriate next step of
management?
A- Give colchicine 12 mg followed by 0.6 mg 12 hours later
B- Prednisolone injection of the first Left MTP
C- Prescribe Colchicine 0.6 mg as needed
D- Prescribe allopurinol 100 mg Life long
E- Oral Prednisolone for three month
10) A 25-year-old woman with SLE gives birth to a baby with complete heart block,
who subsequently requires pacemaker insertion. Which one of the following
antibodies is most likely present in the mother that correlates with her baby’s illness?
A- Anti-centromere antibody
B- Anti-ds DNA
C- Anti-histone antibody
D- Anti-Ro/SSA antibody
E- Anti-smith antibody
11) A 36-year-old woman presents with 2-months history of small joints pain and
swelling, these are affecting her wrists and mainly MCPs bilaterally. Her right ankle
and few of her MTP joints are also painful with episodic swellings. She denies any
fever or skin rash but very bothering mouth ulcers. However, she used to have
recurrent throat infections until recently, her father has psoriasis. She was pregnant 4
times with 2 grown up children and 2 abortions before the age of 10 weeks. She has
definite polyarthritis on MSK exam. Which one of the following laboratory test are
you going to consider as initial step in evaluating this patient?
A- Anticardiolipin antibodies, Lupus anticoagulant and Anti-B2-glycoprotien
antibodies
B- Anti-ds DNA antibodies, Anti-Smith antibodies, Anti-RO and Anti-LA antibodies
C- Antinuclear antibodies, Rheumatoid factor and Anti-citrullinated peptide
antibodies
D- Anti-neutrophilic cytoplasmic antibodies (c-ANCA and p-ANCA)
E- Viral serology: parvovirus-B19 IgM, EBV-IgM, HBsAg, HbsAb, Anti-HCV-
antibodies
12) 25-year-old man is evaluated for a 3-year history of low back and bilateral
buttock Pain that has gradually increased over the past year. The pain is worse in the
morning and after inactivity, he feels better after stretching his back. He has 90
minutes of morning stiffness in his back. Ibuprofen provides moderate relief of
symptoms. He reports no other arthritic symptoms, rash, or gastrointestinal
symptoms. Family history is notable for his paternal uncle with longstanding back
problems. On physical examination, vital signs are normal. There is painful and
diminished forward flexion and extension of the lumbar spine. Tenderness to
palpation over both buttocks is noted. Laboratory studies reveal an erythrocyte
sedimentation rate of 35 mm/h, HLA-B27 testing is negative. Plain radiographs of
the lumbar spine and sacroiliac joints are normal. Which of the following is the most
appropriate diagnostic test to perform next?
A- CT of the sacroiliac joints
B- MRI of the sacroiliac joints
C- Technetium bone scan
D- Ultrasonography of the sacroiliac joints
E- Reassurance and advise for 2 weeks of bedrest
13) A 31-year-old man is evaluated In the emergency department for fever and red
painful eyes. He reports a 1-month history of intermittent painful oral and genital
ulcers, knee pain, and fever and 1 week of bilateral eye redness, pain and
photophobia. On physical examination, temperature is 38.3 and pulse rate is
100/min. Slit lamp examination reveals white cells in the anterior chambers of both
eyes. There are aphthous ulcers on the tongue and one aphthous ulcer on the
scrotum. Swelling and warmth are noted in both knees. The heart, lung, abdominal,
and neurologic examinations are normal. Which of the following is the most likely
diagnosis?
A- Bechet syndrome
B- Crohn's disease
C- Sarcoidosis
D- Systemic lupus erythematosus
E- Spondyloarthritis
14) A 35-year-old female patient present with signs and symptoms of inflammatory
heel pain for 6 months and no sacroiliac joint pain, patient has uveitis controlled
with local steroid there is erosion in x Ray of Achilles tendon insertion. Which of the
following diagnostic test confirm diagnosis?
A- ACPA
B- HLA B-27
C- ANA
D- ESR/CRP
E- ANCA
15) A 34-year-old female known case of SLE pregnant for 7 weeks, which of the
following is true:
A- Cyclophosphamide should be avoided due to fetal congenital malformations
B- Steroids should be stopped
C- Methotrexate can be continued during pregnancy
D- Steroids teratogenicity is more than methotrexate
E- Hydroxychloroquine should be stopped because it’s teratogenic
16) 26-year-old woman presents with 2-months history of pain and swelling in the
hands. She has daily morning stiffness lasting 3-4 hours. She is 4 months postpartum
with uncomplicated pregnancy. She has no history of rash and is otherwise well. She
took ibuprofen but without sufficient relieve of her symptoms. On examination: her
vital signs are as follows: Temp 37, BP 110/68, Heart rate 82/min, RR 16/min. She
has her 2nd and 3rd PIPs and MCPs and the wrists tender and swollen bilaterally.
Initial laboratory findings reveal normal CBC and elevated CRP 15 mg/dl (normal
<3). Which of the following is the most likely diagnosis?
A- Inflammatory osteoarthritis of the hands
B- Rheumatoid arthritis
C- Psoriatic arthritis
D- Crystal-induced arthritis
E- Parvovirus B19 infection
17) Patient with pain in his PIP and MCP with bilateral swollen wrist for 4 weeks he
has mild pain in the right upper quadrant with elevated liver enzymes, what is the
most likely diagnosis?
A- Autoimmune hepatitis
B- Hepatitis B induced arthritis
C- Primary sclerosing cholangitis
B
18) A 27-year-old man diagnosed with Ankylosing spondylitis 2-year ago on
physiotherapy and naproxen. In the past 6-month he experience increased pain and
fitness. One month ago, the naproxen was discontinued and he was started on
indomethacin with no improvement. In physical examination, the patient had severe
sacroiliac joint tenderness, Schober test 12 cm and ESR 40. What is the appropriate
next management?
A- Methotrexate
B- Adalimumab
C- Sulfasalazine
D- Rituximab
19) A 65-year-old man was admitted 3 days ago to coronary care unit with acute MI
and was treated with thrombolytic therapy and was started on low dose Aspirin,
metoprolol and valsartan. This morning, he developed acute and sudden left knee
pain and swelling. His cardiac status is control. His vital signs are stable. Left knee
exam showed moderate to severe effusion with limited range of motion and
tenderness of the knee joint line. The rest of the MSK exam was unremarkable. What
is the best investigation modality to reach your final diagnosis?
A- CBC and Blood culture
B- Serum ANA, RF and anti-CCP antibodies
C- Knee joint aspiration and synovial fluid analysis
D- Knee X-ray
20) 45-year-old woman with Crohn's disease is referred to the rheumatology clinic
for the evaluation of a new onset joints pain that starts 2 weeks ago. Her history is
significant for a long-standing Crohn's disease that remains persistently active
despite treatment, so she was start recently on anti-TNF a (infliximab) infusion. On
examination: she has active synovitis of small joints of both hands (PIPs and MCPs)
and both elbows with decreased range of movements. Her labs showed: WBC= 2.8,
HGB= 13, platelets= 5, ESR= 60, normal renal and liver function tests. RF +ve, ANA=
1:320. Which one of the following is the most likely diagnosis of this patient?
A- Arthropathy associated with inflammatory bowel disease
B- Drug induced lupus
C- Reactive arthritis
D- Rheumatoid arthritis
E- Palindromic rheumatism
00) 70-year-old patient has lost weight in the last months, with signs and symptoms
of GCA, he lost his vision, when the headache resolved, his vision gradually came
back. What is the next step:
A- Temporal artery biopsy
B- Steroids 5 mg for 3 days
C- Steroids 1 mg per kg
D- NSAIDs
C
Orthopedic
21) Which part in axillary view X-Ray can help in determining the direction of
shoulder dislocation?
A- Coracoid process
B- Acromion
C- Glenoid
D- Clavicle
22) A 30-year-old patient was involved in motor vehicle crash and sustained fracture
dislocation of C6-7. On examination, you expect all the following findings EXCEPT:
A- Weak shoulder abduction
B- Weak fingers flexion
C- Absent sensation over the ulnar side of the hand
D- Absent triceps reflex
23) All of the following causes of advanced hip arthritis in a 20-year-old female
except:
A- Avascular necrosis
B- Sickle cell anemia
C- Primary osteoarthritis
D- Neglected DDH
24) Middle aged lady with long history of back pain and leg pain now presented
with severe leg pain and inability to pass urine. Her investigations should include all
of the following except:
A- MRI lumbar spine
B- Rectal exam
C- Admission
D- Urgent gynecology and urology consultation
D
29) Most common organism after orthopedic surgery?
A- Garm +ve
B- Gram -ve
C- Anaerobes
34) A 35-year-old male injured his knee while playing football, he can’t walk on his
knee. What is the examination for extensor mechanism?
A- Ask him to walk even if he can’t
B- Anterior drawer test
C- Schober test
D- Straight leg raises test (active extension of knee)
35) When you can’t see lower cervical vertebra in lateral X-Ray what you will order?
A- Open mouth view
B- Swimmer view
C- Axillary view
36) Differential diagnosis of hip pain in a child include all of the following EXCEPT:
A- Legg-calf-perthes disease
B- Slipped Capital Femoral Epiphysis
C- Septic arthritis of the hip
D- Osgood-Schlatter Disease
37) All of the following muscle are part of the rotator cuff muscle EXCEPT:
A- Supraspinatus
B- Infraspinatus
C- Subscapularis
D- Teres major
D
38) A patient with valgus knee was seen in orthopedic OPD, which knee ligament
will be stretched:
A- Medial collateral ligament
B- Medial meniscus
C- Lateral collateral ligament
D- Lateral meniscus
39) During your call duties at hospital, the nurse called you to see patient, surgery
was performed hip fixation in a 70-year-old male about 4 hours ago, he was having
difficulty breathing, oxygen saturation was 90% in room air, vital signs is stable,
what is your differential diagnosis?
A- Acute atelectasis
B- Pulmonary edema secondary to fluid overload
C- Post-operative pneumonia
D- Spinal shock because patient had epidural anesthesia septic shock due to surgical
site infection
40) A 40-year-old women complaining of (...) lower back pain for 1 week she is
neurologically intact (everything normal) MSK exam also normal, no bladder
symptoms with no history of trauma, the patient known case of osteoarthritis..
A- Non-operative treatment and close follow up
B- Referral to spinal clinic
C- MRI of the spin
D- Bone scan
A
Patient c/o choking every time he eats, where is the tumor :
-Piriform fossa
-Subglottic area
Pt with knee trauma complaining of locking and joint line pain with positive MacMurray test,
which of the following has no effect on operation decision:
-Site of the injury
-Type of the injury
-Duration of the symptoms. (not sure)
-The amount of the effusion
Pt with posterior compartment syndrome of the leg which of the following is true regarding this
patient:
- hypertension is a risk factor
-Intracompartment pressure >30 is a cutt-off for the diagnosis
-Coagulation is a risk factor
- The will increase with passive planter flexion
- diabetes is a risk factor
7 yo with congenital horner syndrome, what is most characteristic manifestation regarding this
disease :
-Iris heterogeneous
- ptosis
-Miosis
- anhydrosis
- amblyopia
- A 5 year old male with recurrent otitis media. He has this procedure done (see picture) you
are following him now as the primary care physician.
what is the most common complication of this procedure?
A. Dislodgment
B. Pain
C. hearing loss
D.Otorrhea
Persistent perforation
An 55-years old man came to your office complaining of right nasal obstruction with recurrent
epistaxis, see his PNS CT ()ﻧﻔس اﻟﻠﻲ ﻓﻲ اﻟﺳﻼﯾدات
A. MRI
B. Biopsy
C. FESS
D. FNA
E. Maxillectomy
A 34 year old female patient has difficulty in hearing for few years now, Her ear exam is
essentially normal. This is the audiogram:
which is the best recommendation?
A. Hearing Aids
B. Antibiotics for 10 days
C. Myringotomy & Grommets insertion
D. Stapedectomy
E. MRI
Posterior knee fracture PT can’t do dorsflextion
L4L5 اﻟﺟواب
Patella fracture treatment
ORIF
2 2 years child with learning difficulties, otoscope exam showed dull TM , which of the following
is the most important investigation:?
-Pure tone audiometry
-Tempanogram
-Mastoid xray
-Nystagmogram
Palpable gab in the knee knee
Primary tendon repair
What is the type of thoracolumbar injury is least associated with neurological deficit?
-fracture dislocation
-chance fracture
-burst fracture
-compressor fracture
-hangman fracture
Cannot planterflex, posterior knee dislocation.
Tibial nerve injury
Cervical protection?
Hard collar
10 cm open fracture with mild contamination what is the gastello classification:
Type II
You are a resident at alnoor hospital called to assess traumatic patient with GCS 11 and need to
clear cervical injury which of following statements is correct:
- ask the patient if there is a neck pain, if not remove the collar
- examine the for neurological deficit if not remove the collar
- order AP, lateral views with open mouth radiograph if clear remove the collar
- Do not remove the collar now wait for later
- Order MRI
65 year old lady with long standing hx of advanced knee osteoarthritis and associated
degenerative medial meniscal tear who is known to have well controlled DM and HTN,
presented with knee pain, limited ROM, tender joint line. What is the definitive treatment?
a. arthroscopic partial menisectomy
b. high tibial osteotomy
c. knee fusion
d. total knee replacement
40.20 years old male basketball player, had an injury to his shoulder while he was trying to stop
another player scoring, and you were in ER when patient arrives to hospital, what is the best
xray view to determine the direction of shoulder dislocation is:
A- Shoulder AP view
B- Shoulder Axillary view
C- Shoulder Scapular view
D- Shoulder CT scan
A patient with valgus knee was seen in orthopedic OPD, which knee ligament will be
stretched:
A- Medial collateral ligament
B- Medial meniscus
C- Lateral collateral ligament
D- Lateral meniscus
43.20 years old female playing football, she had direct injury to her knee from another player,
she couldn't continue playing, her knee was swollen, patient CAN NOT extent the knee fully,
you are suspecting either ACL injury or quadriceps. what is the main function of the
Quadriceps muscle:
A- Knee active flexion
B- Knee varus stabilizer
C- Knee active extension
D- Knee valgus stabilizer
52-year-old man known case of chronic gout and hypertension on thiazides 25 mg once
daily, comes to your clinic with one day history of pain in right big toe with associated swelling.
He denies any history of trauma or other joints involvement. His BP is 125/70 with normal rest
of his vital signs. He has swollen and tender big toe. His labs show normal CBC, ESR 40 CRP 9,
serum uric acid 10 mg/dL and creatinine 2.8 mg/dL.
Which of the following is the best modality for acute gout management in this patient:
A. Naproxen 500 mg twice daily
B. Colchicine 0.5 mg q8 hourly
C. Systemic or local steroid injection
D. High protein Diet restriction
E. Ice and rest
A 30-years old woman diagnosed with SLE 1 year ago, presents to rheumatology clinic for
follow up, She complains of a new onset joints pain and excessive hair loss. She states that she
is
taking her medications regularly.
on examination: her B.P is 150/90 mmHg (she was not hypertensive before), other vital signs
are
normal. she has a non-scarring alopecia and tenderness in wrist joints of both hands with
reduction of wrist extension.
Here are some of her labs results: WBC- 6, HGB= 11, Platelets= 110. Normal renal and liver
function test. Urinalysis= Protein 1, RBC casts ++.
Which of the following parameters are considered as indicators of lupus flare in this
patient?
A. Active urinary sediments and thrombocytopenia
B. Arthralgias and non- scarring alopecia
C. High titers of anti-ds DNA and normal complements levels
D. Low complements 3 and 4 level, high anti-ds DNA titers, active urinary sediments
E. Newly diagnosed hypertension
A 40-year-old woman is evaluated for a 6-month history of pain and swelling in her left
thumb, left fifth finger, and left foot. She has a 4-year history of lumbar and thoracic back pain
that is worse with bending and lifting and sometimes awaken her from sleep. She also has
morning stiffness lasting 2 to 3 hours. Naproxen is only mildly helpful for the pain.
On physical examination, vital signs are normal. Patches of erythema and scaling behind the
right ear and on the scalp at the occiput are noted. Fusiform swelling of the left thumb and left
fifth finger is present. There is mild lumbar tenderness, and full range of motion of the lumbar
spine and cervical spine is noted. No other joint swelling or tenderness is present.
Laboratory studies, including complete blood count with differential, comprehensive metabolic
panel, rheumatoid factor, and urinalysis, are normal; HLA-B27 testing is positive.
Which of the following is the most likely diagnosis?
A) Ankylosing spondylitis
B. Inflammatory bowel disease-associated arthritis
C. Psoriatic arthritis
D. Reactive arthritis
E. Adult-onset Still's disease
53. A 36-year-old woman presents with 2 months hx of small joints pain and swelling, These are
affecting her wrists and mainly MCPs bilaterally. Her right ankle and few of her MTP joints are
also painful with episodic swellings. She denies any fever or skin rash but very bothering mouth
mouth ulcers. However, she she used to have recurrent throat infections until recently, her
father
has psoriasis. She was pregnant 4 times with 2 grown up children and 2 abortions before the
age
of 10 weeks. She has definite polyarthritis on MSK exam.
Which one of the following laboratory test are you going to consider as initial step in
evaluating this patient?
A. Anticardiolipin antibodies. Lupus anticoagulant and Anti-B2-glycoprotien antibodiesse
B. Anti-ds DNA antibodies, Anti-Smith antibodies, Anti-RO and Anti-LA antibodies (not sure)
C. Antinuclear antibodies, Rheumatoid factor and Anti-citrullinated peptide antibodies
d. anti-neutrophilic cytoplasmic antibodies (c-ANCA and p-ANCA)
E. Viral serologie: parvovirus-B19 IgM, EBV-IgM, HBsAg, HbsAb, Anti-HCV-antibodies.
A 32-vear-old man known case of hypertension on calcium channel blocker presents to your
clinic complaining of 2 days duration of only right ankle pain and swelling. The pain started
suddenly and became severe within few hours with inability to walk and bear weight. He denies
history of preceding trauma, fall or recent infection. He had similar attack 2 years ago in the left
ankle that resolved gradually over 1 week. His physical exam showed swelling and redness of
the ankle joint with limited range of motion passively and actively. What is the most likely
diagnosis?
A. Systemic lupus erythematosus
B. Rheumatic Fever
C. psoriatic Arthritis
D. Rheumatoid Arthritis
E. Acute Gouty Attack
What’s true about DM NP. retinopathy.? ﻣوﺟود ﻓﻲ اﻟﺗﺟﻣﯾﻌﺎت اﻟﻘدﯾﻣﮫ
cannot lead to blindness
A 66 year old elderly male presents with heart disease and has sudden loss of vision in one
eye, fundus examination reveals a cherry red spot, the most important investigation to do is :
a. CT brain
b. Brain angio
C. Carotid doppler
d.Ocular ultrasound
---------------------------------------------------------------------------------------------------------------------------
Veniger eye ﻣوﺟود ﻓﻲ اﻟﺗﺟﻣﯾﻌﺎت اﻟﻘدﯾﻣﺔ
irrigation and ER
Cataract common cause? ﻣوﺟود ﻓﻲ اﻟﺗﺟﻣﯾﻌﺎت اﻟﻘدﯾﻣﺔ
HTN
DM اﻟﻣﻔروضaging ﻟﻛن اﻟﺧﯾﺎرات ﻛﺎﻧت ﻏﯾر
--------------------------------------------------------------------------------
55 years old male known case of open angle glaucoma, which of the following statement is
more appropriate in describing his condition
a. the angle is congenitally abnormal
b) ??? is an effective treatment
C. can be treated surgically
d. is treated with drops only
Where's the defect,
occipital lope
Ear pain and all examination normal most common referral
Tempomandibular referred pain( not sure)
Ear itching ?
Suction microscopy and eardrops fungal
A nine months old baby has put a plastic bead in his ear. On otoscopy, it is present in deeper
portion of meatus and totally occluding the meatus. The best way to remove this bead is:
Quadriceps function,
extention of knee
30 y o. Unable to bend his back two days, test to comfirm? اﻟﻜﯿﺲ ﻛﺎﻧﺖ ﺗﯿﺒﯿﻜﺎلAS
Straight leg rise
Modified chopper test
Compression test
After shoulder dislocation in 50 years old male, you are suspecting rotator cuff tear. which
muscle is responsible of shoulder internal rotation:
A- Deltoid Muscle
B- Supraspinatus Muscle
C- Subscapularis Muscle
D- Infraspinatus Muscle
years old female presenting with paralysis of 3rd
, 4th and 6th nerves with involvement of
ophthalmic division of 5th nerve, this localizes the lesion to:
a- Cavernous sinus thrombosis
b- Parietal lobe
C- Frontal lobe
d- Base of skull
Management of acute angle glocoma
Lazer irodotomy
Lasek
Immolsificatoon
Mineers disease , two attacks
_____________________________________________________________
55 years old patient present with neck mass (photo of bangali man)
FNA was preformed Sqaumous cell carcinoma
What is next step
CT Brain
CT Trachea
Panendoscopy
Incisional biopsy
Excisional biopsy
______________________________________________________________________________
this is an Xray of6 year old boy, presented with fever, throat pain then started to become
stridrous. This is his lateral soft tissue X ray.
what is the best therapeutic recommendation?
A. Reassurance
B. Observation
C. Antibiotic for two weeks.
D. Admission, humidified O2, steroids& IV Antibiotics
E. Admission & tracheostomy
_______________________________________________________________________
1 السؤال
1. 75 y.o male presented to ER with painless sudden loss of vision OD, and he is a known case of
hypertension.
Your working diagnosis is
A. Ischemic optic neuropathy
B. Optic neuritis
C. Papilledema
D. Optic atrophy
1 درجات
2 السؤال
1. A 72-year-old man presents with an acutely painful right knee. On examination, he has a temperature
of 37C with a hot, swollen right knee. Of relevance amongst his investigations, was his white cell
count which was 12.6 Χ 10 9 /l and a knee X-ray revealed reduced joint space and calcification of the
articular cartilage. Culture of aspirated fluid revealed no growth. What is the most likely
diagnosis?
A. Gout
B. Pseudogout
C. Psoriatic monoarthropathy
D. Rheumatoid arthiritis
E. Septic arthritis
1 درجات
3 السؤال
1. 35 y.o female presenting to you with unilateral lid ptosis, the most likely cause will be
A. Hyperthyroidism
B. hypothyroidism
1.
C. thyroid eye disease
E. No contraindication
1 درجات
5 السؤال
1. 25 y.o female presenting to ER with hypopyon, this can be the presenting symptom of
A. corneal ulcer
B. keratoconus
C. pterygium
D. kertoglobus
1 درجات
6 السؤال
1. 60 y.o male with uncontrolled diabetes HBA1C is 13, his ophthalmologist told him that he has non-
proliferative diabetic retinopathy, which of the following is more likely to lead to vision loss in
his condition
A. Macular degeneration
B. Vitritis
C. Retinal vasculitis
D. Macular edema
1 درجات
7 السؤال
1. (55 years) old male patient presented to outpatient clinic with right shoulder pain for over three
months, X-ray right shoulder was normal and you are suspecting rotator cuff tendinitis, he was
treated conservatively with anti-inflammatory medication. Three months later at his follow up
appointment you noticed weakness in shoulder abduction and rotation, what is the test of choice to
rule out rotator cuff tear:
A. Shoulder CT scan
B. Shoulder MRI
C. Shoulder ultra sound
D. Shoulder X-ray with contrast
E. Shoulder X-ray in 3 views
1 درجات
8 السؤال
1. A 27-year-old woman is evaluated for a sudden onset of joint pain and significant stiffness in her
fingers, wrists, knees and ankles for the past 5 days. She had a brief fever with muscle aches a few
days ago and a faint pink rash on her arms and legs at the same time. She reports no other symptoms.
She works at a daycare. On physical examination, there is mild warmth and tenderness of the first
through fifth proximal interphalangeal and metacarpophalangeal joints bilaterally as well as the
wrists.
Which of the following will most likely confirm the diagnosis?
A. Antinuclear antibodies
B. HIV testing
C. Parvovirus B19 testing
D. Rheumatoid factor
E. Anti-CCP antibodies
1 درجات
9 السؤال
1. 17 y.o female presenting to ER with eye pain after minor trauma to the eye upon examination you
find that she has corneal abrasion, Patching an eye to promote healing of corneal abrasion is
contraindicated in:
A. Contact lens wearer with abrasion.
B. Corneal ulcer.
C. Fungal keratitis.
D. all of the above.
1 درجات
10 السؤال
1. 50 year old male patient hospitalized with a diagnosis of community acquired pneumonia. What is
the most effective therapy for this patient?
A. Intravenous Methylprednisolone
B. Inhaled Salbutamol
C. Inhaled Budesonide
D. Intravenous Ceftriaxone and Azithromycin
E. Oral Dextromethorphan (cough suppressant)
1 درجات
11 السؤال
1. What is the first-line agent for treatment of symptomatic bradycardia?
A. Atropine
B. Lidocaine
C. Epinephrine
D. Vasopressin
1 درجات
12 السؤال
1. A 53-year-old woman with no past medical history, comes to the emergency department with 4-day
history of palpitation. She also complains of fatigue and dyspnea. Her BP is 153/72 and HR is 137
bpm, O2 sat was 95% on room air. On physical examination shows an irregularly irregular rhythm.
Auscultation reveals no crackles.
Which of the following is the best next step in management of this patient?
A. Adenosine
B. Metoprolol
C. Amiodarone
D. Heparin
E. Synchronized cardioversion
1 درجات
13 السؤال
1. 32-year-old female falls from the 10th floor of her apartment building in an apparent suicide attempt.
Upon presentation, the patient has obvious head and extremity injuries. Primary survey reveals that
the patient is totally apnic. By which method is the immediate need for a definitive airway in
this patient best provided?
A. Orotracheal intubation
B. Nasotracheal intubation
C. Percutaneous cricothyroidotomy
E. LMA
1 درجات
14 السؤال
1. A 57-year-old man has right side weakness. His blood pressure is 220/140. Funduscopic exam
demonstrates arteriolar narrowing and arteriovenous nicking. Neurological exam revealed decrease
in power and sensation in upper and lower limbs. His past medical history is significant for
hyperglycemia, hypertension, and gout. His medications are losartan 50 mg daily,
hydrochlorothiazide 25 mg daily, metformin 500 mg and allopurinol 100 mg daily. He has no
medication allergies. His labs within normal limits. Which of the following is an appropriate
treatment option?
A. Nitroglycerin IV
B. Amlodipine orally
C. Sodium nitroprusside IV
D. Nimodipine orally
E. Labetalol IV
1 درجات
15 السؤال
1. A 35-year-old male presents with a painful ,red area in his right arm for 4 days. It has a well
demarcated border .This condition is most commonly caused by which of the following organisms?
A. Escherichia coli
B. Haemophilus influenza
C. Staphylococcus aureus
D. Streptococcus pyogenes
E. Staphylococcus albus
1 درجات
16 السؤال
1. Which of the following statements is true of infants with gastroschisis?
A. It is associated with malrotation and unhealthy bowel
1 درجات
17 السؤال
1. ( 30 years ) old lady involved in motor vehicle accident, she sustained tibia fracture and has 2 CM
open laceration over the fracture site, it is clean and no contamination, what will be the treatment
of choice :
A. Close reduction and above knee cast
B. Close reduction and below knee cast
C. Irrigation and debridement, close reduction and Intra medullary nail
1 درجات
18 السؤال
1. A 62-year-old woman presents complaining of hand pain bilaterally that has been gradually
progressive over the past 2 years. She has previously worked in a factory making gloves for more
than 35 years. You suspect osteoarthritis. All of the following factors on history or physical
examination are characteristic of this diagnosis EXCEPT:
A. Evidence of bilateral swelling and warmth affecting the wrists only
B. Joint space narrowing and osteophytes at the proximal and distal interphalangeal joints on x-
ray
C. Pain that becomes worse when preparing meals
D. Presence of Heberden’s nodes
E. Stiffness that lasts for a few minutes after brief periods of rest with occasional locking of the
more affecting joints
1 درجات
19 السؤال
1. A 52-year-old man comes to the emergency department because he has had vomiting, nausea, and
abdominal pain for the past 12 hours. He says he attempted suicide 3 days ago by "taking everything
in the medicine cabinet." He was stuporous for approximately 12 hours after the overdose but felt
better the following day. At this time, he has jaundice and pain in the right upper quadrant.
Overdose of which of the following drugs is most likely to have caused the pain, vomiting, and
jaundice in this patient?
A. Acetaminophen
B. Aspirin
C. Cimetidine
D. Diphenhydramine
E. Triazolam
1 درجات
20 السؤال
1. A 42-year-old obese diabetic man presents with 5 x 5 cm painful dark red colored swelling in the
upper back with multiple openings discharging pus. This condition is most likely a consequence
of which of the following?
A. An infection of hair follicles
B. An insect bite
C. Severe sunburn
D. A subcutaneous injection of a medication
E. Spreading cellulitis
1 درجات
21 السؤال
1. What is the next step in treating patient with right shoulder pain ( Sub acromion
impingement) failed full course of anti-inflammatory medication and intense physical therapy
program, shoulder MRI shows tendon inflammation with no sign of acute cuff tear:
A. Give additional anti-inflammatory medicine on top of what patient is taking
1 درجات
22 السؤال
1. A 55-year-old woman is evaluated for an 18-month history of increasingly severe knee pain with the
inability to arise when seated on the floor. She does not have pain at rest or nocturnal pain.
Medications are celecoxib and omeprazole.
On physical examination. vital signs are normal. Knee exam reveals bilateral joint line tenderness
with no warmth, erythema, or swelling.
Plain antero-posterior knee radiographs show mild medial joint space narrowing, osteophytes; there
are no erosions or osteopenia.
Which or the following is the most appropriate management?
A. Glucosamine supplements
B. Knee replacement surgery
C. Physical therapy
D. Prednisone
E. Change celecoxib to ibuprofen
1 درجات
23 السؤال
1. A 75 year old male presents with nasal obstruction for 2 months, His nasal endoscopy showed a
right sided nasal mass.
This is his PNS CT. What is the best next step?
C. FESS
D. Biopsy
E. Observation
1 درجات
24 السؤال
1. 19-year old man presents to ER after motor vehicle crash (MVC). Patient is disoriented and
complaining of severe chest pain in the right side. During primary survey, his O2 saturation is 91%
on room air, no breath sound on the left with subcutaneous emphysema. Heart rate is 120 beat/min
but blood pressure in 80/60. What is the most likely cause of this hemodynamic status?
A. Tension pneumothorax
B. Open pneumothorax
C. Simple pneumothorax
D. Massive pneumothorax
E. Occult pneumothorax
1 درجات
25 السؤال
1. 42-year-old male with a history of alcoholism presents to the hospital with confusion, diplopia
(ophthalmoparesis), unsteady gait, and nystagmus.
What is the most likely cause of these symptoms?
A. Thiamine deficiency
B. Hypoglycemia
C. Stroke
D. Hepatic encephalopathy
E. Hypothyroidism
1 درجات
26 السؤال
1. A 20 year old male, with a medical history significant for active intravenous drug user presented to
emergency room complaining of persistent fever. Pertinent positive on physical examination are
temperature of 39.3°C, multiple pink, painful nodules on the palmar surface of both hands and grade
3 pansystolic murmur on fourth left parasternal area. Three sets of blood culture are sent.
What is the most likely cause of persistent fever?
A. Infective endocarditis
B. Lung abscess
C. Acute rheumatic fever
1 درجات
27 السؤال
1. A 20 year old young man had long standing foul smelling ear discharge. Recently he has developed
otalgia, fever, headache and vomiting.
What is his most likely diagnosis?
A. Brain abscess
B. Bezolds abscess
C. Perilabyrinthine fistula
D. Otitis externa
E. Meningitis
1 درجات
28 السؤال
1. 33-year-old female, came to ER after she fell down the stairs. She is unconscious, hypotensive and
hypoxic. What would be the best initial intervention to improve her oxygen saturation?
A. High flow nasal cannula
B. Face mask
C. Bilevel positive airway pressure (BiPAP)
D. Endotracheal intubation
E. Emergency tracheostomy
1 درجات
29 السؤال
1. A 2.8-kg. neonate with excessive salivation develops respiratory distress. Attempts to pass an
orogastric catheter fail because the catheter coils in the back of the throat. A chest film is obtained
and shows right upper lobe atelectasis and a normal gas pattern in abdomen. The most likely
diagnosis is:
A. Congenital diaphragmatic hernia
B. Proximal esophageal atresia with a distal tracheoesophageal (TE) fistula
C. Pyloric stenosis
D. Duodenal atresia
E. Inguinal hernia
1 درجات
30 السؤال
1. A 50-year-old man has brain tumor with increased intracranial pressure. Clinical examination
reveals convergent squint of the right eye. The patient is unable to rotate it externally.
Which of the following nerves is the single most likely affected?
A. abducent
B. maxillary
C. oculomotor
D. ophthalmic
E. trochlear
1 درجات
31 السؤال
1. No history of trauma, based on CT brain findings what is your next step?
B. MRI brain
C. MRI and MRA (angiography)
D. CTA (angiography)
E. Conventional angiography
1 درجات
32 السؤال
1. A 34-year-old woman with a history of depression presents to the emergency department with
tachycardia, drowsiness and shallow breathing. After her initial presentation, she begins to become
confused and has a seizure. An ECG is performed and shows QT prolongation and ventricular
arrhythmias. An arterial blood gas shows metabolic acidosis.
An overdose of which of the following drugs is the most likely cause of patient symptoms?
A. Acetaminophen
B. Amitriptyline
C. Diazepam
D. Digoxin
E. Methanol
1 درجات
33 السؤال
1. 60-year-old poorly controlled diabetic patient presents to clinic with otalgia purulent ear discharge
for the last 3 weeks. On examination the right ear shows mucopurulent discharge and granulation
. What is the most likely cranial nerve to be affected in this condition?
A. Trigeminal nerve
B. Facial nerve
C. Glossopharyngeal nerve
D. Vagus nerve
E. Optic nerve
1 درجات
34 السؤال
1. A 35-year-old woman has epigastric pain due to peptic ulcer for years. He suddenly feels severe pain
in the epigastrium. Examination reveals rigidity in the epigastrium (board-like rigidity)
Which of the following structures is the single most likely penetrated by the above ulcer?
A. greater sac
B. left kidney
C. liver
D. pancreas
E. spleen
1 درجات
35 السؤال
1. 70 years old female presented to the OPD with history of right calf intermittent claudication for six
months. Claudication distance has been stable at 300 meters and relieved at rest. She is known
diabetic and hypertensive with Hb1c of 11%. Patient had Lower limbs CT angiography which
showed 80% right common femoral artery occlusion and 70% right popliteal artery occlusion
with antegrade filling of both anterior and posterior tibial arteries. Her best option for
treatment at this stage would be:
A. Right common femoral artery endarterectomy
B. Right femoral popliteal artery bypass using reversed saphenous venous graft
C. Optimize medical treatment (control BP and DM and start on ASA and Pentoxifylline)
1 درجات
36 السؤال
1. 8 y.o child with orbital cellulitis, the most important step in your management is
A. Admit patient to start iv antibiotics
B. start oral antibiotics
C. treat in outpatient clinic
D. order plain X- rays of the orbit as part of the investigations
1 درجات
37 السؤال
1. A 65-year-old woman has a longstanding history of primary hypothyroidism with poor compliance to
treatment. She presents to ER with decreased level of consciousness and you are concerned of her
having myxedema coma.
Which of the following abnormality will support your working diagnosis?
A. Hypernatremia
B. Hypercapnia
C. Hypercalcemia
D. Hyperthermia
1 درجات
38 السؤال
1. A 68-year-old man with congestive heart failure presents to the emergency department with
dehydration. He has been vomiting and had diarrhea for the past 36 hours. His blood pressure is
105/75 mm Hg with a pulse of 62 beats/minute. His blood digoxin level is 4.1 ng/ml (normal 1.0 to
2.6).
Which of the following is most effective in treatment of the patient condition?
A. Deferoxamine
B. Fab fragment antibodies
C. N-acetylcysteine
D. Protamine sulphate
E. Sodium bicarbonate
1 درجات
39 السؤال
1. A 30-year-old man has bleeding from the nose. On examination; the bleeding comes from the
anterior inferior area of the septum.
Which of the following arteries is the major contribution of blood in this area?
A. anterior ethmoidal
B. greater palatine
C. posterior ethmoidal
D. sphenopalatine
E. superior labial
1 درجات
40 السؤال
1. 68 years old male patient presented to the OPD clinic with cramping left thigh and calf pain for 8
months. The pain increases with walking and relived by rest for 15 min. He came to the OPD because
the pain has been more frequent and comes at rest. The patient Has history of DM, HTN,
Hypercholesteremia, and he quit smoking 5 years ago. In your evaluation all of the following
findings are expected EXCEPT:
A. Absent left posterior tibial pulse
B. Aortic and lower limbs CT angiography showed 90% Left external iliac artery occlusion
1 درجات
41 السؤال
1. ( 70 years ) old female patient, had a fall at home and sustained displaced femoral neck fracture, the
best treatment choice is:
A. Bed rest and traction till fracture heals then mobilization
1 درجات
42 السؤال
1. A 14 year old female had a common cold 6 weeks ago. This was followed by left forehead pain, upper
eyelid edema and a temperature of 38.5C. Later her fever rose to 40.5C, the lid edema increased and
she started to complain of decrease vision. On examination the eye showed orbital swelling and
proptosis.
What is the best initial image?
A. Sinus X-Ray
B. CT PNS
C. MRI brain
D. US orbits
E. MRI PNS
1 درجات
43 السؤال
1. A 67-year-old British man is brought to the emergency department in a stable condition with a
history of haematochezia. He has no relevant previous medical or drug history, works as a builder, is
a non-smoker and drinks 60 units of alcohol per week.
What is the most likely cause of gastrointestinal bleeding (GIB)?
A. Variceal bleeding is the most likely cause.
B. Peptic ulcer disease is the cause of bleeding.
C. Variceal bleeding is the commonest cause of bleeding in Egypt and India
D. The bleeding will have originated distal to the ligament of Treitz
E. His predicted mortality attributable to this illness is higher than that of an inpatient who
develops UGIB
1 درجات
44 السؤال
1. regarding same patient above in previous question
How would you treat corneal abrasion?
A. Using topical antibiotics
D. Using latanoprost
1 درجات
45 السؤال
1. 65 year old male presents with hearing loss. On exam he has ipsilateral Unilateral secretary otitis
media and painless neck mass. What is his most likely diagnosis?
A. AIDS
B. Parapharyngial tumor
C. Glandular fever
D. Nasopharyngeal carcinoma
E. Hypopharyngeal carcinoma
1 درجات
46 السؤال
1. A 33-year-old woman is evaluated for concerns about the development of rheumatoid arthritis. She
has some mild joint discomfort in the hands at the end of the day. She has been smoking five
cigarettes per day for the past 6 years. Her identical twin sister was recently diagnosed with
seropositive rheumatoid arthritis. Her physical examination is unremarkable.
Which of the following is the most appropriate preventive measure for this patient at
risk for developing rheumatoid arthritis?
A. Avoid food/drinks containing high-fructose syrup
B. Begin hydroxychloroquine
C. Begin probiotics
D. Smoking cessation
E. Modification of work
1 درجات
47 السؤال
1. You were called to manage a 35-year male motor vehicle accident victim who has sustained multiple
leg fractures in addition to splenic and liver lacerations few days ago.
The patient developed sudden shortness of breath and central chest pain worsening with deep
breathing. No associated cough, wheeze or fever. His legs had been in casts from mid tibia till above
knee. BP 90/60, HR 130 regular, Temp 37, RR 32, Oxygen Saturation 88% on room air. Chest
examination revealed central trachea, symmetrical air entry without added sounds. CVS exam
showed elevated JVP with loud pulmonary component of second sound. Leg exams showed diameter
difference at mid thigh showing larger left leg. Laboratory work up showed drop in hemoglobin from
10 to 8 and stable WBC of 12000 in addition to normal cardiac enzymes and chest X ray .
The most appropriate next step is :
A. D dimer testing
B. Doppler US both legs
C. Echocardiogram
D. Start anticoagulant empirically
1 درجات
48 السؤال
1. A 1-year-old female has a palpable mass within one of her labia majora. Radiographic examination
reveals that a loop of intestine has herniated.
Which is the single most likely the diagnosis?
A. direct inguinal hernia
E. lumbar hernia
1 درجات
49 السؤال
1. A 54-year-old man has a 3-hour history of chest pain and dyspnea. He is 1-week post colectomy for
colon cancer. On examination, Temp 37.5°C, BP 110/60 mmHg, HR 130/min, and RR 24/min. Oxygen
saturation is 88% on room air and 94% on oxygen, 4 L/min. Cardiac examination shows tachycardia.
Breath sounds are normal. Serum creatinine concentration is 3 mg/dl (normal 0.84-1.2 mg/dl) and
his complete blood count and coagulation profile is normal. CT pulmonary angiography shows left
sided pulmonary embolism.
Which of the following is the most appropriate immediate treatment?
A. Unfractionated heparin
B. Rivaroxiban
C. Enoxaparin
D. Fondaparinux
E. Dabigatran
1 درجات
50 السؤال
1. 30-year-old restrained driver was involved in a motor-vehicle crash. He is hemodynamically stable
and has a large seat belt sign on the abdomen. His abdomen is tender to palpation. In this patient one
should be most concerned about:
A. Liver and spleen injury
B. Transection of the head of the pancreas
C. Pelvic fracture
D. Hollow-viscus injuries
E. Abdominal wall contusion
1 درجات
51 السؤال
1. A 65 male with history of HTN and DM seen in the clinic with new diagnosis of a-
fib. Hemodynamically stable. All investigations normal. Which of the following is the next step?
A. Aspirin + metoprolol
B. Warfarin + metoprolol
C. Cardioversion
D. Amiodarone
E. Cardiology referral
1 درجات
52 السؤال
1. A 60-year old woman was found on routine screening to have iron
deficiency anemia. Her other labs parameter and physical Examination was unremarkable. Which
one of the following is the most appropriate management of the patient?
A. prescribe an iron supplement
B. arrange for the patient to undergo colonoscopy
1 درجات
53 السؤال
1. A 63 year-old diabetic man is brought to the emergency room in severe discomfort and pain. He was
last able to urinate 14 hours ago. On examination, he has a suprapubic tender, globular (spherical)
mass. BP 140/90mmHg, HR 96/min, RR 22/min, T 36.6 oC.
Which of the following is the most appropriate next step?
A. Urinalysis
B. Alpha blockers
C. Oral antibiotics
D. Abdominal ultrasound
1 درجات
54 السؤال
1. A 70 year old man admitted with right frontal intracerebral hemorrhage. On day 3 he developed
generalized seizure that aborted after 10 minutes by two doses of intravenous benzodiazepines.
What is the most appropriate next step in managements?
A. Intravenous valproic acid loading followed by maintenance
B. Oral carbamazepine
C. Start antiepiletic if second seizure developed
D. Start midazolam infusion
E. Start oral antiepileptic if EEG showed an epileptic discharges
1 درجات
55 السؤال
1. 29 years old female 2 weeks postpartum has right upper quadrant pain and vomiting, on
examination she has RUQ tenderness with a +ve Murphy’s sign. Which of the following abdominal
ultrasound finding confirms your diagnosis?
A. Hepatomegaly
B. Gall Bladder stone, thickening of GB & peri-cholecystic fluid
C. Large stone in the common bile duct with dilated common bile duct
1 درجات
56 السؤال
1. You are called by the on-call senior surgical nurse to assess a 74-year-old male patient who has
undergone a right hemicolectomy earlier that same day. You are informed his pulse is 115 BPM and
BP is 90/55. On arrival to the surgical ward, your immediate management should consist of:
A. Request 2 units of blood for transfusion and call a senior member of the surgical team.
B. Arrange for a septic screen and an abdominal and pelvic CT scan with IV and oral contrast.
C. Complete the patient’s vitals and clinical assessment and commence resuscitations measures
as appropriate.
D. Consent the patient for surgical re-exploration and arrange with operating room staff for
immediate transfer.
E. Review the patient’s medical records and obtain a full history from the patients relative.
1 درجات
57 السؤال
1. 44 y.o male presented to ER complaining of pain in his left eye associated with headache, nausea,
vomiting and red eye.
Your most likely treatment is
A. Phacoemulsification
B. Laser photocoagulation
C. Laser iridotomy
D. LASIK
1 درجات
58 السؤال
1. A 25-year-old man presented with 2 weeks history of on and off fever associated with sore throat
and fatigability. His vitals are normal apart from T 39 C. His blood work revealed the following:
2.
3. Which of the following is the single most likely diagnosis?
A. Acute kidney injury
B. Acute gout
C. Leukemoid reaction
D. Spontaneous tumor lysis syndrome
E. Thrombotic thrombocytopenic purpura
1 درجات
59 السؤال
1. A 20-year-old woman has 48-hour history of progressive headache and fever that are associated with
nausea, vomiting, and epigastric pain. On examination, she appears restless and confused. Temp 38.1
C, pulse 110 bpm, BP 150/90mmHg, and RR 22/min. remainder of physical examination is
unremarkable. Laboratory studies show Hb 9.7g/dl, MCV 102 fL, WBC 6 X10 9/L, PLT 5 X109/L,
haptoglobin 10mg/dl, LDH 600U/l, and creatinine 1.4mg/dl. A peripheral blood film shows
fragmented RBCs. Direct Anti-globulin test is negative.
Which of the following is the most likely diagnosis?
A. Aplastic anemia
B. Disseminated intravascular coagulation
C. Thrombotic thrombocytopenic purpura
D. Autoimmune hemolytic anemia with immune thrombocytopenic purpura
1 درجات
60 السؤال
1. A 25-year-old man is evaluated for the gradual onset of bilateral low back pain without radiation to
the lower extremities daily, with increasing severity over the past year. The pain now awakens him
during the night 2 to 3 times per week, with morning stiffness lasting more than an hour. His pain
improves with exercise. He takes ibuprofen which provides mild relief.
On physical examination, vital signs are normal. Limited lateral bending bilaterally and a reduction in
forward flexion at the lumbar spine are noted. The remainder of the examination is normal.
Which of the following is the most appropriate diagnostic test to perform next?
A. Bone scan
B. CT of the lumbar spine
C. MRI of the sacroiliac joints
D. Radiography of the hip joints
1 درجات
61 السؤال
1. A 23-year-old male patient with Crohn’s disease develops an enterocutaneous fistula.
The metabolic abnormality likely to occur is:
A. Hypocalcemia
B. Hypochloremia
C. Hypokalemia
D. Hypophosphatasemia
E. Hyperamylasemia
1 درجات
62 السؤال
1. A 35-year old male, presented to emergency room with fever and cough. He was well until 3 days
earlier, when he suffered the onset of nasal stuffiness, mild sore throat, and a cough productive of
small amounts of clear sputum. Patient smokes one pack of cigarettes daily. Today, he decided to
seek physician assistance because of an increase in temperature to 38.3°C and worsening cough.
Chest X-ray showed right lower lobe consolidation. What is the most likely diagnosis?
A. Acute bronchitis
B. Community acquired pneumonia
1 درجات
63 السؤال
1. The best treatment choice of Distal radius fracture in 65 years old male, minimal displacement on X-
ray is:
A. Open reduction and k-wire fixation
B. Close reduction and above elbow cast application
1 درجات
64 السؤال
1. A 60-year-old man, who has COPD with frequent exacerbations and glucocorticoid treatment,
presents to ER with 1-day history of increasing fatigue, nausea, and vomiting. On examination, he is
afebrile, HR 120 bpm, BP 97/75mmHg, and O2 saturation 92% on room air. He has dry mucous
membranes and diffusely tender abdomen without guarding or rebound tenderness. His
investigations reveal hyponatremia and evidence of UTI on urine analysis.
Which of the following is the most appropriate diagnostic test?
A. Random serum ACTH and cortisol
1 درجات
65 السؤال
1. A 35 year old man with bad type-1 diabetes presents with a pressure of 65. His anterior chamber is
deep but you find neovascularization everywhere, in the retina and on the iris. What do you think is
causing the pressure rise:
A. Acute angle closure glaucoma
1 درجات
66 السؤال
1. 55 y.o male known case of open angle glaucoma, which of the following statement is more
appropriate in describing his condition?
A. the angle is congenitally abnormal
B. moxifloxacin is an effective treatment
1 درجات
67 السؤال
1. ( 25 years ) old basketball player, sustained left shoulder dislocation at the field while playing, you
ordered X-ray and confirmed anterior shoulder dislocation, the preferred method of treatment is:
A. Open reduction under general anesthesia in operating room
B. Close reduction under general anesthesia in operating room
C. Close reduction under conscious sedation in ER
D. Close reduction under conscious sedation in ER with vital signs monitoring
1 درجات
68 السؤال
1. A 71 year-old diabetic man presents to the Emergency Room complaining of scrotal pain. On
examination, he had a small area of purplish denuded skin, with surrounding erythema and
tenderness. A CT scan shows some air within the scrotum and perineum. BP 90/50mmHg, HR
110/min, RR 24/min, T 39.6 oC.
Which of the following is the most essential step to perform?
17 4-10.5 x 109 /L
WBC
APTT 28 22-37 s
A. Scrotal elevation
B. IV immunoglobulin
C. Surgical debridement
D. Incision and drainage
E. Administer intravenous antibiotics
1 درجات
69 السؤال
1. 45 years old engineer had sudden onset of severe abdominal pain, he smokes one pack per day, on
examination he has normal vital signs except for tachycardia (pulse 100 beat/minute), he has
tenderness on superficial palpation.
Of the following, what is the most likely test to aid the diagnosis of this patient?
A. Flat plate of the abdomen
B. Serum amylase
C. Abdominal series
D. EKG & cardiac enzymes
E. EGD
1 درجات
70 السؤال
1. A 33-year-old male with history of type 1 diabetes who was brought to emergency room with a
history of altered mental status and found to have diabetes ketoacidosis. His PH is 6.8. Which of the
following statements is correct regarding the use of sodium bicarbonate in diabetes
ketoacidosis?
A. Sodium bicarbonate can be added if PH < 6.9
B. Sodium bicarbonate improves mortality in diabetes ketoacidosis
C. The use of Sodium bicarbonate is limited to type 1 diabetes patients
D. Sodium bicarbonate can lead to severe hyperkalemia
E. Sodium bicarbonate is associated with post-treatment metabolic acidosis
1 درجات
71 السؤال
1. A 72-year-old man has had a sensation of retrosternal burning that is worse at night and after meals
for the last 4 months. It partially responds to ‘over the counter’ antacid treatment. Which is the
single most appropriate initial diagnostic investigation?
A. Barium meal
B. Barium swallow
C. CT scan thorax
D. EGD
E. 24h pH studies
1 درجات
72 السؤال
1. A 50 year old male patient presented with otalgia. On examination both external auditory canal and
tympanic membrane are normal.
What is the most likely site of origin of his pain?
A. Temporomandibular joint
B. Subglottic region
C. Nasopharynx
D. Eye
E. Maxilla
1 درجات
73 السؤال
1. A 49-year-old woman presents with acute onset of left hemiplegia and right-side facial weakness,
involving the upper and lower facial movements. Which of the following is the most likely
diagnosis?
A. Right pontine infarct
1 درجات
74 السؤال
1. 30 y male known case of bronchial asthma & insulin dependent DM presenting to ER department
because progressive shortness of breath . He had mild dry cough but no wheeze .In addition he had
frequent loose bowel motions without abdominal pain or fever .The bowel motions didn’t contain
blood nor had black color . on examination BP 90/60 HR 110 Temp 36 RR 26 Oxygen saturation 95%
on room air . JVP 1 cm above sternal angle .Chest and cardiac examination was normal . His lab work
showed:
Na 120
Chloride 95
HCO3 15
PH 7.25
PO2 100
PCO2 30
2.
3. The most appropriate management is :
A. Inhaled bronchodilator & Intravenous steroids
B. Intravenous Insulin
C. Sputum & stool cultures
D. Intravenous fluid bolus & maintenance
E. Chest & Abdominal X rays
1 درجات
75 السؤال
1. A 49-year-old man with history of hypertension presents to the emergency department with acute
onset of right hemiparesis and aphasia. The time he was last seen normal was about 45 minutes prior
to arrival. The National Institutes of Health Stroke Scale (NIHSS) score is 14. Which of the following
is the best next step?
A. Start intravenous tissue plasminogen activator (tPA)
1 درجات
76 السؤال
1. A 31-year-old man is evaluated In the emergency department for fever and red painful eyes. He
reports a 1-month history of intermittent painful oral and genital ulcers, knee pain, and fever and 1
week of bilateral eye redness, pain and photophobia.
On physical examination, temperature is 38.3 and pulse rate is 100/min. Slit lamp examination
reveals white cells in the anterior chambers of both eyes. There are aphthous ulcers on the tongue
and one aphthous ulcer on the scrotum. Swelling and warmth are noted in both knees. The heart,
lung, abdominal, and neurologic examinations are normal.
Which of the following is the most likely diagnosis?
A. Behcet syndrome
B. Crohn’s disease
C. Sarcoidosis
D. Systemic lupus erythematosus
E. Spondyloarthritis
1 درجات
77 السؤال
1. A 40-year-old female with slowly progressive conductive deafness with normal drum & Eustachian
tube function. Her CHL worsened after her last pregnancy. What is the most likely diagnosis?
A. Otitis media with effusion
B. Otosclerosis
C. Malingering
D. Tympanosclerosis
E. Ossicular discontinuity
1 درجات
78 السؤال
1. 65-Year-old Male with histologically confirmed Adenocarcinoma of the Colon presented to the ER
with per rectal Bleeding, upon arrival he was hypotensive 85/60 and tachycardic, what is your
initial Response in the Emergency Room?
A. Staging CT Abdomen and Thorax to determine the TNM status
B. Start chemotherapy as soon as possible
C. Canula Insertion for IV fluids and cross match
D. Immediate CT angiography to localize the Bleeding
E. Immediate Colonoscopy for Tumor coagulation to stop the bleeding
1 درجات
79 السؤال
1. What is your most likely diagnosis?
A. Lunate dislocation
B. Peri lunate dislocation
C. Scaphoid fracture
D. Lunate fracture
E. Distal radio-ulnar joint dislocation
1 درجات
80 السؤال
1. In grade 3A comminuted mid tibia fracture, the soft tissue defect was over the mid shaft tibia and
after fixation with intra medullary nail you were unable to approximate the wound edges and defect
was over 2 CM over the tibia bone, what choice of treatment that is acceptable ?
A. Leave wound open, multiple irrigation and debridement then perform skin graft
B. Immediate skin graft after nail fixation
C. Consult plastic surgeon prior fixation to consider local rotational flap with skin graft
1. 25 years old female presented to ER with sudden loss of vision OD, painful eye movement,
and she is a known case of MS. What is your working diagnosis?
a. Ischemic optic neuropathy
0
b. Optic neuritis
C. Papilledema B
d. Optic atrophy
2. 44 years old male presented to ER complaining of pain in his left eye associated with
headache, nausea, vomiting and red eye. What is your working diagnosis?
a. Open angle glaucoma
b. Congenital glaucoma
C
c. Acute angle closure glaucoma
d. Glaucoma suspect
3. 55 years old male known case of open angle glaucoma, which of the following statement is
more appropriate in describing his condition
a. the angle is congenitally abnormal
b) travaprost is an effective treatment B
C. can not be treated surgically
d. is treated with drops only
4. 35 years old female presenting to you with unilateral lid retraction. What is the most likely
cause?
a. hyperthyroidism
b. hypothyroidism C
C. thyroid eye disease
d. orbital cellulites
5. 8 years old child with orbital cellulitis, the most important step in your management is
a. check and document vision
b. start oral antibiotics
c. treat in outpatient clinic A
d. order plain X-rays of the orbit as part of the investigations
6. 60 years old male with uncontrolled diabetes HBA1C is 13, his ophthalmologist told him
has proliferative diabetic retinopathy, Which of the following is more likely to lead to sever
vision loss in his condition
a- Macular degeneration
b- Vitritis D
C- Retinal vasculitis
d- Retinal detachment
7. A 35 year old man with bad type-1 diabetes presents with a pressure of 65. His anterior
chamber is deep but you find neovascularization every where in the retina and on the iris.
What do you think is causing the pressure rise?
a- Acute angle closure glaucoma
b- Chronic open angle glaucoma
C
C- Neovascular glaucoma
d- Acute attack of hyperglycemia
8. 30 years old male patient with family history of retinal detachment came to discuss risk
factors. Which of the following is a risk factor for retinal detachment?
a- Black race D
b- Male sex
C- Presbyopia
d- Myopia
9. A man calls you complaining of splashed bleach in his eye. You should instruct him to:
a- Patch the eye and immediately go to the emergency
b- Irrigate the eye with clean water and go to the emergency
C- Immediately apply lubricating ointment and then go to the emergency B
d- Immediately wash the eye with high PH solution
10. 35 years old man presenting to ER with headache, dilated right pupil, double vision and right
side ptosis, he is otherwise healthy What is your working diagnosis?
a- 6th nerve palsy
b- 4th nerve palsy D
C- Horner's syndrome
d- cerebral aneurysm
11. 33 years old male referred to eye clinic because he has Anisocoria.
What is the most likely cause?
a- Retinal disease
b- Media opacity D
C- Optic nerve disease
d- 3rd nerve palsy
12. years old female presenting with paralysis of 3rd, 4th and 6th nerves with involvement of
ophthalmic division of 5th nerve, this localizes the lesion to:
a- Cavernous sinus
b- Parietal lobe A
C- Frontal lobe
d- Base of skull
13. A 55 year old diabetic patient gives history of sudden painless loss of vision in one eye for
the past two weeks and there is no history of trauma. On examination the anterior segment is
normal but there is no red reflex, which one of the following is the most likely cause?
a. Vitreous hemorrhage
b. Optic atrophy
A
C. Cataract
d. Acute angle closure glaucoma
n
14. A 30-year-old man came to the outpatient department because he had suddenly developed
double vision. On examination his right eye was found turned medially at rest. The most likely
anatomical structures involved are:
a. Medial rectus and superior division of oculomotor nerve
b. Inferior oblique and inferior division of oculomotor nerve C
c. Lateral rectus and abducent nerve
d. Superior rectus and trochlear nerve
15. A 66 year old elderly male presents with heart disease and has sudden loss of vision in one
eye, fundus examination reveals a cherry red spot, the most important investigation to do is :
a. CT brain
b. Brain angio C
C. Echocardiogram
d. Ocular ultrasound
16. A 5-Years old child with history of fever and swelling of the face ant to the both ears (parotid
gland enlargement). Which is the most common complication?
A. Orchitis
B. Encephalitis
C. Mastoiditis D
D. Meningitis
E. Conjunctivitis
17. A 37-Years old post cholecystectomy male patient came with unilateral face swelling and
tenderness. He has past history of measles when he was young. On examination there is swelling
over the cheek/face. What is the most likely diagnosis?
A. Sjogren Syndrome
B. Parotid cancer
C. Bacterial Sialadenitis C
D. Sarcoidosis
E. Salivary gland stone
18. A 55-Years old male patent. Smoker 1 pack/day. Presented with hoarseness. On exam, there
was a LEVEL-III cervical mass. Which is your next best step?
A. Flexible Laryngoscopy
B. CT brain
C. CT neck A
D. FNA Neck Mass
E. Biopsy
19. A 6-Years old girl is brought to the family health center by her mother. The child today had
sudden onset of a painful sore throat, difficulty swallowing, headache and abdominal pain. She
was exposed to someone at school that recently was diagnosed with a "strep throat". On
examination the child has a temperature of 40°C. She has tender anterior cervical lymph nodes
and exudative tonsils. What treatment would you offer for this child?
A. Zithromax
B. Penicillin B
C. Ciprofloxacin
D. No antibiotics, rest, fluid, & acetaminophen
E. Trimothoprim
20. A 5-Years old male with recurrent Otitis Media. Has this procedure done.
You are following him now as his primary care. This can stay in place without
any Sequala for?
A. Six months
B. Twelve months
C. Eighteen months
D. Twenty Four months D
E. Thirty months
22. This is an X-ray of 6 years old boy, presented with fever, throat pain
then started to be stridrous. This is his lateral soft tissue X-Ray. What is
the best therapeutic recommendation?
A. Reassurance, and follow up in OPD
B. Observation in ER for 2 hours
C. Antibiotics for two weeks
D! Admission, Humidified O2, Steroids, & IV Antibiotics
E. Admission & Tracheostomy using adult size tube
D
23. An 18-Years old young man came to your office complaining for
recurrent throat infections & snoring. He complain also of occasional
nasal regurgitation when drinking liquids. Which procedure is
considered contra-indicated?
A. Uvulo-Palato-Pharyngo-Plasty (UPPP)
B. Tonsillectomy C
c. Adenoidectomy
D. Uvula excision
E. Tongue reduction
24. a 58 years old alcoholic and smoker. You find this clinical finding.
What is the most appropriate next step?
a. CT of brain
b. CT of trachea
c. FNA biopsy C
d. excisional biopsy
e. incisional biopsy
25. a 34 year old doctor presented with rcurrent episodes of vertigo lasting hours. With fullness,
tinnitus, hearing loss of the left ear. No associated neurological symptoms, increasing frequency;
twice during important meetings. Once while driving. What investigations will you order to
confirm the diagnosis?
a. audiogram
b. ENG A
c. MRI
d. CT temporal bone
e. OAE
26. a 4 year old child was playing with his friends, he developed sudden spell of choking, cough
and went blue, he settled down and left with dry cough. What is the management?
a. chest xray
b. start antibiotic C
c. do bronchoscopy
d. give bronchodilators
e. give cough suppressors
27. 4 year old presents with a history of URTI that progress to fever, severe throat pain, drooling
and respiratory distress within hours, the child is toxic and assumes a setting poseture with the
chin up and mouth open. What organism is responsible for his condition?
a. Staph aureus
b. klebsiella pneumoniae C
c. hemophilus influenza
d. parainfluenza
e. Moraxella catarrhalis
28. A 20 year old man had long standing foul smelling ear discharge. Recently he has developed
otalgia, fever, headache and vomiting. What serious condition you should suspect?
A. Brain abscess
B. Bezolds abscess
C. Perilabyrinthine fistula E
D. Otitis externa
E. Meningitis
29 A 7 year old boy has history of snoring and apnea, you suspect adenoid hypertrophy. What is
the best diagnostic test?
A. Chest x-ray
B. Lateral soft tissue X-ray of the neck C
C. Naso-Pharyngeal Endoscopy
D. Anterior rhinoscopy by nasal speculum
E. Tympanogram
30.A 25-Years old male presented with 10-day history of nasal obstruction, nasal discharge, and
headaches with low-grade fever. In the last 2 days he started to have right periorbital swelling
redness. What is the most appropriate management?
A Incision and drainage of the right periorbital abscess
B. Referral of the patient to the neurosurgeon C
C. CT scan of the paranasal sinuses and IV antibiotics
D. CBC and blood culture.
E. MRI of the paranasal sinuses
31. 30 year old male was involved in a car accident and sustained posterior knee dislocation of
the Rt side. The patient is unable to dorsiflexed his Rt. Foot, this is most likely due to:
A- Deep proneal nerve injury
B- Sciatic nerve injury A
C- L4-5 disc herniation
D- Peripheral neuropathy
32. You were on call at Al-Noor Hospital and were called to see a 30-year old male patient who
was a driver involved in road traffic accident. He was driving at 140 Km/hr and lost control of
the car because he was under the influence of alcohol. The car rolled over 8 times and the patient
was ejected from the car and found 10 meters from his car. When he was brought to the
emergency department with a cervical collar and his Glasgow Coma Scale is 11, the trauma team
asked you to clear his cervical spine. What is your best action?
a. ASK the patient and if he does not have neck pain, remove the cervical collar
b. Examine the patient and if he is neurologically intact, remove the cervical collar
D
C. Do AP, Lateral & Open mouth views of the cervical spine and if there were normal, remove
the cervical spine
d. You cannot remove the cervical spine now because the patient is alcohol intoxicated
33. a 25 year old male fell on out stretched hand and is complaining of wrist pain.
Anteroposterior and lateral xray of the wrist showed undisplaced distal radial fracture with NO
shortening, angulation or displacement. The tx of choice of such injury is:
a. application of cast
b. application of external fixator
c. open reduction and internal fixation A
d. closed reduction and percutaneous k-wire fixation
34. 42 year old man is brought in ER after a 10 meter fall off a ladder. Physical examination
shows slightly deformed lower extremity with a 5 cm soft tissue defect over the anterolateral
aspect of his leg, the wound appears relatively clean with no gross contaminants present.
Radiograph depict a short oplique proximal one third diaphyseal tibial fracture. What is his
Gustillo classification?
a. I
b. II B
c. IIA
d. IIB
35. 35 year old female k/c of chronic renal failure on dialysis for 8 years presented to ER with
severe right knee pain and swelling after she fell off while coming down the stairs, she is unable
to weight bear, cant do active knee extension and have a palpable gap anterior to her patella, The
best treatment option of this pt is:
a. cast immobilization for 4-6 weeks B
b. primary tendon repair
c. ligament reconstruction
d. Rest, Ice, compression and elevation followed by physical therapy
36. 40 year old teacher presented with 6 months hx of neck pain that radiated to the lt upper limb
with numbness and tingling sensation. She has decrease sensation over the thumb and ring finger
and 4/5 wrist extension. What is the cause of this patient symptoms? I
a. lt sided C6-7 cervical disc herniation
b. lt sided carpal tunnel syndrome A
c. lt sided C7-T1 cervical disc herniation
d. do odontoidum
37. 65 year old lady with long standing hx of advanced knee osteoarthritis and associated
degenerative medial meniscal tear who is known to have well controlled DM and HTN,
presented with knee pain, limited ROM, tender joint line. What is the definitive treatment?
a. arthroscopic partial menisectomy
b. high tibial osteotomy D
c. knee fusion
d. total knee replacement
38. 24 year-old male had multiple Injuries after a fall from 17 m height, what is the best method
to protect the thoracolumbar spine while waiting for radiological investigation?
A- Bed rest
B- Spine board
c- Log rolling C
D- Brace
39. After shoulder dislocation in 50 years old male, you are suspecting rotator cuff tear. which
muscle is responsible of shoulder internal rotation:
A- Deltoid Muscle
B- Supraspinatus Muscle
C
C- Subscapularis Muscle
D- Infraspinatus Muscle
40.20 years old male basketball player, had an injury to his shoulder while he was trying to stop
another player scoring, and you were in ER when patient arrives to hospital, what is the best
xray view to determine the direction of shoulder dislocation is:
A- Shoulder AP view B
B- Shoulder Axillary view
C- Shoulder Scapular view
D- Shoulder CT scan
41.30 years old male was doing hand wrestling with other friend, he felt sudden onset elbow and
upper forearm pain, when you was the patient there was huge ecchymosis and bruises in all
forearm, if you are suspecting distal Biceps rupture which joint motion weakness you expecting
in clinical exam:
A- Elbow extension
B- Elbow flexion
C- Forearm supination
C
D- Forearm pronation
42. A patient with valgus knee was seen in orthopedic OPD, which knee ligament will be
stretched:
A- Medial collateral ligament
B- Medial meniscus A
C- Lateral collateral ligament
D- Lateral meniscus
43.20 years old female playing football, she had direct injury to her knee from another player,
she couldn't continue playing, her knee was swollen, patient CAN NOT extent the knee fully,
you are suspecting either ACL injury or quadriceps. what is the main function of the
Quadriceps muscle:
A- Knee active flexion
B- Knee varus stabilizer C
C- Knee active extension
D- Knee valgus stabilizer
44.25 year-old had a frontal collision to a wall by his car & suffered from multiple injuries and
you that he is unable to move his both lower limbs with loss of sensation. Which of the
following thoracolumbar injury pattern carry the highest risk of this finding?
A- Fracture dislocation
B- Burst fracture
C- Chance fracture
D- Compression fracture
A
45. 12 hours after ORIF for forearm fractures in a 20 year old patient which was uneventful, you
were called to evaluate the patient who is having severe pain that is out of proportion to the
Surgical procedure, not relieved by narcotics. The volar compartment is hard and painful to
palpation. Your evaluation is highly suggestive of volar compartment syndrome. Which of the
following is true regarding the patient's condition:
A- Hypotension is a risk factor A
B- Intra compartmental Pressure over 40 mmHg is the cutoff for the diagnosis
C. The patient is expected to have pain with passive fingers flexion
D- Coagulation disorders are not risk factor
46. A 26-year-old woman presents with 2-months history of pain and swelling in the hands. She
has daily morning stiffness lasting 3-4 hours. She is 4 months postpartum with uncomplicated
pregnancy. She has no history of rash and is otherwise well. She took ibuprofen but without
sufficient relieve of her symptoms.
On examination: her vital signs are as follows: Temperature 37, Blood pressure 110/68, Heart
rate 82/min, respiratory rate 16/min. She has her 2nd and 3rd PIPs and MCPs and the wrists
tender and swollen bilaterally. a
Initial laboratory findings reveal normal CBC and elevated C-reactive protein 15 mg/dl (normal
<3). Which of the following is the most likely diagnosis?
A. Inflammatory osteoarthritis of the hands
B. Rheumatoid arthritis
C. Psoriatic arthritis
B
D. Crystal-induced arthritis
E. Parvovirus B19 infection
47. a 51 Year-old man has been complaining of arthralgia, myalgias, fever, and a 10-kg weight
loss months. For the past 2 weeks, he has noted an erythematous rash on his legs and andial
crampy
Y abdominal pain. He has also noted numbness in his right foot for the past few days.
On physical examination, Temperature 37.8° C. BP 150/95 mmHg. There are skin ulcers over
the lower extremities. His chest is clear. He has LLQ abdominal tenderness. There is no
testicular tenderness. The al muscles are tender. His joints are tender to palpation without
synovitis. Weakness of the right foot to dorsiflexion is noted.
Laboratory investigations reveal Hb 101 g/L, ESR 85mm/Hr, CRP 25 mg/L, Creatinine 120
umol/ AST 55 IU/L, ALT 60 IU/L and Urinalysis +2 Protein, 3+ blood, no casts.
Which of the following procedures is most likely to confirm the diagnosis?
A. Esophagogastroduodenoscopy
B. Mesenteric and renal arteriography Vasculitis B
C. Punch biopsy of the skin Neuropathy
D. Testicular biopsy
E. Nerve conduction study
Fan
48. A 25- years old woman with SLE gives birth to a baby with complete heart block, who
subsequently requires pacemaker insertion. Which one of the following antibodies is most
likely present in the mother that correlates with her baby's illness?
A. Anti-centromere anti body
B. Anti-ds DNA D
C. Anti-histone antibody
D. Anti-Ro/SSA antibody
E. Anti-Smith antibody
49. A 65-year-old man was admitted 3 days ago to coronary care unit with acute MI and was
treated with thrombolytic therapy and was started on low dose Aspirin, metoprolol and valsartan.
This morning, he developed acute and sudden left knee pain and swelling. His cardiac status is
under control. His vital signs are stable. Left knee exam showed moderate to severe effusion
with limited range of motion and tenderness of the knee joint line. The rest of the MSK exam
was unremarkable. What is the best investigation modality to reach your final diagnosis?
A. CBC and Blood culture
B. Serum uric acid level.
C. Serum ANA, RF and anti-CCP antibodies
D
D. Knee joint aspiration and synovial fluid analysis
E. Knee X-ray
50. A 22-year-old man comes to emergency department with history of dyspnea and hemoptysus
5 hours prior to presentation. These symptoms were associated with recurrent attacks of oral
ulcers and chronic on and off painful eyes with redness that improve with topical steroid, He also
provided history of scarring scrotal area, His oxygen saturation is 87% on room air and his
respiratory rate is 33 breathes per minute. Which of the following is most likely diagnosis?
A. Takayasu's Arthiritis
B. Giant cell arthiritis
I C Granulomatosis Polyangitis D
D Behcet's Disease
E. Cogan's Syndrome
51. A 31 year-old woman presents with complaints of fatigue and pain in her both hands and feet
thaat interferes with her daily activities for the last 3 months, on and off evening rise of
temperature (37.8 C-38 C). She is taking paracetamol tablets which partially improve her pain
and fever. She also noticed skin rash over sun exposed areas and on her face. On further history
taking: she mentions that one month ago she had a right sided pain that worsens with coughing
and sneezing, and improved by NSAIDs.
Findings on examination: a red rash over both cheeks and bridge of the nose, synovitis of MCPs
and PIPs of both hands, MTPs of both signs are normal as well as normal systemic examination.
Her CBC showed: WBC=3.8 (N 4-11) Hb: 12.5 (N= 12-15), Platelets= 184 (N= 150-400).
Which one of the following treatment options will you start for patient based on her
current disease status?
A. Azathioprine, NSAIDs, small dose aspirin
B Small dose prednisolone, hydroxychloroquine and continue NSAIDs
C. Methotrexate, hydroxychloroquine, prednisolone
B
D. NSAIDs with high dose prednisolone
E. Short course of oral and topical steroids
52/A 32-vear-old man known case of hypertension on calcium channel blocker presents to your
clinic complaining of 2 days duration of only right ankle pain and swelling. The pain started
suddenly and became severe within few hours with inability to walk and bear weight. He denies
history of preceding trauma, fall or recent infection. He had similar attack 2 years ago in the left
ankle that resolved gradually over 1 week. His physical exam showed swelling and redness of
the ankle joint with limited range of motion passively and actively. What is the most likely
diagnosis?
A. Systemic lupus erythematosus
B. Rheumatic Fever E
C. psoriatic Arthritis
D. Rheumatoid Arthritis
E. Acute Gouty Attack
53. A 36-year-old woman presents with 2 months hx of small joints pain and swelling, These are
affecting her wrists and mainly MCPs bilaterally. Her right ankle and few of her MTP joints are
I
also painful with episodic swellings. She denies any fever or skin rash but very bothering mouth
mouth ulcers. However, she she used to have recurrent throat infections until recently, her father
has psoriasis. She was pregnant 4 times with 2 grown up children and 2 abortions before the age
of 10 weeks. She has definite polyarthritis on MSK exam.
Which one of the following laboratory test are you going to consider as initial step in
evaluating this patient?
A. Anticardiolipin antibodies. Lupus anticoagulant and Anti-B2-glycoprotien antibodiesse
B. Anti-ds DNA antibodies, Anti-Smith antibodies, Anti-RO and Anti-LA antibodies C
C. Antinuclear antibodies, Rheumatoid factor and Anti-citrullinated peptide antibodies
d. anti-neutrophilic cytoplasmic antibodies (c-ANCA and p-ANCA)
E. Viral serologie: parvovirus-B19 IgM, EBV-IgM, HBsAg, HbsAb, Anti-HCV-antibodies.
54. A 79-year-old man is evaluated for 2 months history of progressive malaise and weakness,
aching bilateral shoulder and hips and stiffness for 2 hours in the morning. This is associated
with visual blurriness. He recently noted aching in his jaw when chewing. On physical
examination, vital signs are normal. Tenderness and slight swelling over the left temple are
present. Painful and limited range of motion of both hips and shoulder is noted. Laboratory
studies show ESR 85 mm/h.
Which of the following is the most appropriate initial management?
A. CT of the head
B. Low dose aspirin E
C. Methotrexate
D. Temporal artery biopsy
E Prednisolone
55. A 40-year-old woman is evaluated for a 6-month history of pain and swelling in her left
thumb, left fifth finger, and left foot. She has a 4-year history of lumbar and thoracic back pain
that is worse with bending and lifting and sometimes awaken her from sleep. She also has
morning stiffness lasting 2 to 3 hours. Naproxen is only mildly helpful for the pain.
I
On physical examination, vital signs are normal. Patches of erythema and scaling behind the
right ear and on the scalp at the occiput are noted. Fusiform swelling of the left thumb and left
fifth finger is present. There is mild lumbar tenderness, and full range of motion of the lumbar
spine and cervical spine is noted. No other joint swelling or tenderness is present.
Laboratory studies, including complete blood count with differential, comprehensive metabolic
panel, rheumatoid factor, and urinalysis, are normal; HLA-B27 testing is positive.
Which of the following is the most likely diagnosis?
A) Ankylosing spondylitis
B. Inflammatory bowel disease-associated arthritis C
C. Psoriatic arthritis
D. Reactive arthritis
E. Adult-onset Still's disease
56. A 30-years old woman diagnosed with SLE 1 year ago, presents to rheumatology clinic for
follow up, She complains of a new onset joints pain and excessive hair loss. She states that she is
taking her medications regularly.
on examination: her B.P is 150/90 mmHg (she was not hypertensive before), other vital signs are
normal. she has a non-scarring alopecia and tenderness in wrist joints of both hands with
reduction of wrist extension.
Here are some of her labs results: WBC- 6, HGB= 11, Platelets= 110. Normal renal and liver
function test. Urinalysis= Protein 1, RBC casts ++.
Which of the following parameters are considered as indicators of lupus flare in this
patient?
A. Active urinary sediments and thrombocytopenia
B. Arthralgias and non- scarring alopecia
D
C. High titers of anti-ds DNA and normal complements levels
D. Low complements 3 and 4 level, high anti-ds DNA titers, active urinary sediments
E. Newly diagnosed hypertension
Is
57. A 52-year-old man known case of chronic gout and hypertension on thiazides 25 mg once
daily, comes to your clinic with one day history of pain in right big toe with associated swelling.
He denies any history of trauma or other joints involvement. His BP is 125/70 with normal rest
of his vital signs. He has swollen and tender big toe. His labs show normal CBC, ESR 40 CRP 9,
serum uric acid 10 mg/dL and creatinine 2.8 mg/dL.
Which of the following is the best modality for acute gout management in this patient:
A. Naproxen 500 mg twice daily
B. Colchicine 0.5 mg q8 hourly C
C. Systemic or local steroid injection
D. High protein Diet restriction
E. Ice and rest
58. A 45-year old woman with Crohn's disease is referred to the rheumatology clinic for the
evaluation of a new onset joints pain that starts 2 weeks ago. Her history is significant for a long
standing Crohn's disease that remains persistently active despite treatment, so she was start
recently on anti-TNF a (infiliximab) infusion.
On examination: she has active synovitis of small joints of both hands (PIPs and MCPs) and both
elbows with decreased range of movements. Her labs showed: WBC= 2.8, HGB= 13, platelets=5
ESR= 60, normal renal and liver function tests. RF +ve, ANA= 1:320.
Which one of the following is the most likely diagnosis of this patient?
A. Arthropathy associated with inflammatory bowel disease
B. Drug induced lupus B
C. Reactive arthritis
D. Rheumatoid arthritis
E. Palindromic rheumatism
59. A 25-year-old man is evaluated for a 3-year history of low back and bilateral buttock Pain
that has gradually increased over the past year. The pain is worse in the morning and after
inactivity: he feels better after stretching his back. He has 90 minutes of morning stiffness in his
back. Ibuprofen provides moderate relief of symptoms. He reports no other arthritic symptoms,
rash, or gastrointestinal symptoms. Family history is notable for his paternal uncle with long-
standing back problems.
On physical examination, vital signs are normal. There is painful and diminished forward flexion
and extension of the lumbar spine. Tenderness to palpation over both buttocks is noted.
Laboratory studies reveal an erythrocyte sedimentation rate of 35 mm/h; HLA-B27 testing is
negative. Plain radiographs of the lumbar spine and sacroiliac joints are normal.
Which of the following is the most appropriate diagnostic test to perform next?
A. CT of the sacroiliac joints
I
B. MRI of the sacroiliac joints
C. Technetium bone scan AY B
D. Ultrasonography of the sacroiliac joints
E. Reassurance and advise for 2 weeks of bedrest
60. A 42-year-old woman is evaluated for a 6-month history of pain and swelling of several
small
hand joints, an elbow, and an ankle. She gets modest relief with naproxen. She has no other
medical problems and takes no additional medications.
On physical examination, vital signs are normal. There is tenderness to palpation and swelling of
the second and third proximal interphalangeal joints bilaterally, second and firth
metacarpophalangeal joints bilaterally, left wrist, right elbow, and right ankle. The remainder of
the physical examination is normal.
Laboratory studies are significant for a rheumatoid factor of 85 U/mL and positive anti-cyclic
citrullinated peptide antibodies. Radiographs of the hands and wrists show periarticular
osteopenia at the metacarpophalangeal joints and a marginal erosion at the right second
metacarpal head.
Which of the following is the most appropriate initial treatment?
A. Leflunomide
B. Methotrexate
C. Rituximab B
D. Tofacitinib
E. Adalimumab
6th year Summative exam - 1st term
Q11: a 65 y.o male with hearing loss. O/E he has ipsilateral secretary otitis media and painless
neck mass. What is his most likely diagnosis?
A. AIDS
B. Parapharyngial tumer
C. Glandular fever D
D. Nasopharyngeal carcinoma
E. Hypopharngeal carcinoma
Q13: a 20 y/o young man had long standing foul smelling ear discharge, recently he has
developed otalgia, fever, headache and vomiting. What is his likely diagnosis?
A. Brain abscess
B. Bezolds abscess
C. Perilabyrinthine fistula E
D. Otitis externa
E. Meningitis
Q14: a 50 y/o male pt. Presented with otalgia. O/E both external auditory canal and tympanic
membrane are normal. What is the most likely site of origin of his pain?
A. Temporomandibular joint A
B. Subglottic region
C. Nasopharynx
D. Eye
E. Maxilla
Q16: a 40 y/o female with slowly progressive conductive deafness with normal drum & ET
function. Her CHL worsened after her last pregnancy. What is the most likely diagnosis?
A. OME
B. Otosclerosis
C. Malingering
p
D. Tympanosclerosis
E. Ossicular discontinuity
Q17: 60 y/o poorly controlled diabetic patient presents to clinic with otalgia purulent ear
discharge for the last 3 weeks. O/E the right ear shows mucopurulent discharge and granulation.
What is the most likely cranial nerve to be affected in this condition?
A. Trigeminal
B. Facial
C. Glossopharyngeal
D. Vagus
E. Optic
Q18: 55 y/o male pt. Presented to OPD with right shoulder pain for over 3 months, x-ray right
shoulder was normal and you are suspecting rotator cuff tendonitis. He was treated
conservatively with NSAID. 3 months lated at his follow up appointment you noticed weakness
in shoulder abduction and rotation. What is the test of choice to rule out rotator cuff tear?
A. Shoulder CT
B. Shoulder MRI B
C. Shoulder US
D. Shoulder x-ray with contrast
E. Shoulder x-ray in 3 views
Q19: 30 y/o lady involved in MVA, she sustained tibia fx and has 2 cm open laceration over the
fx site, it’s clean and no contamination. What will be the treatment of choice?
A. Close reduction and above knee cast
B. Close reduction and below knee cast
C. Irrigation and debridement, close reduction and intramedullary nail
D. Irrigation and debridement, open reduction with plate and screws
E. Open reduction with external fixation & debridement
Q20: 25 y/o basketball player, sustained left shoulder dislocation at the field while playing, you
order x-ray and confirmed anterior shoulder dislocation. The preferred method of treatment is:
A. Open reduction under GA in OR
B. Close reduction under GA in OR
D
C. Close reduction under conscious sedation in ER
D. Close reduction under conscious sedation in ER with vital signs monitoring
E. Open reduction under regional anesthesia
Q21: 70 y/o female pt. Had a fall at home and sustained displaced femoral neck fx. The best
treatment choice is:
A. Bed rest and traction till fx heals then mobilization
B. Close reduction and dynamic hip screw
C. Open reduction and dynamic hip screw
D. Hip hemi arthroplasty
E. Skeletal traction as she is a high risk patient
Q22: in grade 3A comminuted mid tibia fx, the soft tissue defect over the mid shaft tibia and
after fixation with intramedullary nail you were unable to approximate the wound edges and
defect was over 2 cm over the tibia bone . What choice of treatment that is acceptable:
A. Leave wound open, multiple irrigation and debridement then perform skin graft
B. Immediate skin graft after nail fixation
C. Insult plastic surgeon prior fixation to consider local flap with skin graft
D. Close wound and defect with tension suture
E. Primary closure using skin metallic clips
Q23: what is the next step in treating patient with right shoulder pain (subacromion
impengement) failed full course of NSAID and intense physiotherapy program, shoulder MRI
shows tendon inflammation with no sign of acute cuff…..
A. Give additional NSAID on top o what pt. Is taking
B. Corticosteroid subacromion injection
C. Corticosteroid shoulder injection
D. Corticosteroid acromion clavicular AC joint injection
E. Surgical intervention
Q24: The best treatment choice of distal radius fx in 65 y/o male, minimal displacement on x-ray
is:
A. Open reduction and K-wire fixation
B. Close reduction and above elbow cast application
C. Close reduction and K-wire fixation
D. Application of mini wrist external fixation
i
E. NSAID & physiotherapy
Q25: a 72y/o man presented with an acutely painful right knee. O/E he has a T=37c with a hot,
swollen right knee. Of relevance amongst his investigations, was his WBC=12. 6*109/L and knee
x-ray revealed reduced joint space and calcification of the articular cartilage. Culture of aspirated
fluid revealed no growth . What is the most likely diagnosis?
A. Gout
B. Pseudogout
C. RA
D. Prosaic monoarthropathy
E. Septic arthritis
Q26: A 25-year-old man is evaluated for the gradual onset of bilateral low back pain without
radiation to the lower extremities daily, with increasing severity over the past year. The pain now
awakens him during the night 2 to 3 times per week, with morning stiffness lasting more than an
hour. He has improvement with exercise and no improvement at rest. He takes ibuprofen with
some improvement.
On physical examination, vital signs are normal. Limited lateral bending bilaterally and a
reduction in forward flexion at the lumbar spine are noted. The remainder of the examination is
normal.
An anteroposterior plain radiograph of the pelvis and sacroiliac joints is unremarkable.
Which of the following is the most appropriate diagnostic test to perform next?
A. Bone scan
B. CT of the lumbar spine
C. MRI of the sacroiliac joints
D. Radiography of the hip joints
E. All of the above
I
Q27: a 62 y/o woman presents complaining of hand pain bilaterally that has been gradually
progressive over the past 2 years. She has previously worked in a factory making gloves for
more than 35 years. You suspect osteoarthritis. All of the following factors on history or physical
examination are characteristic of his diagnosis EXCEPT:
A. Evidence of bilateral swelling and warmth affecting the wrists only
B. Joint space narrowing and osteophytes at he proximal and distal interphalangeal joints on
x-ray
C. Pain that becomes worse when preparing meals
D. Presence of Hebeden’s nodes
E. Stiffness that lasts for a few minutes after brief periods of rest with occasional locking of
the more affecting joints
Q28: a 33y/o woman is evaluated for concerns about the development of RA. She has some mild
joint discomfort in the end of the day. She has been smoking five cigarettes per day for the past 6
years. Her identical twin sister was recently diagnosed with seropositive RA. Her physical
examination is unremarkable. Which of the following is the most appropriate preventive measure
for this patient at risk for developing RA?
A. Avoid food/ drinks containing high fructose syrup
B. Begin hydroxychloroquine
C. Begin probiotic
oD. Smoking cessation
E. Modification of work
Q29: a 27y/o woman is evaluated for a sudden onset of joint pain and significant stiffness in her
fingers, wrists, knees, and ankles for the past 5 days. She had a brief fever with muscle aches a
few days ago and a faint pink rash on her arms and legs at the same time. She reports no other
symptoms. She works at daycare. O/E there is mild warmth and tenderness of the first through
fifth proximal interphalangeal and metacarpophalangeal joints bilaterally as well as wrists.
Which of the following will most likely confirm the diagnosis?
I
A. Antinuclear antibodies
B. HIV testing
C. Parvovirus B19 testing
D. RF
E. Anti-CCP antibodies
Q30: A 31-year-old man is evaluated In the emergency department for fever and red painful eyes.
He reports a 1-month history of intermittent painful oral and genital ulcers, knee pain, and fever
and 1 week of bilateral eye redness, pain and photophobia.
On physical examination, temperature is 38.3 and pulse rate is 100/min. Slit lamp examination
reveals white cells in the anterior chambers of both eyes. There are aphthous ulcers on the tongue
and one aphthous ulcer on the scrotum. Swelling and warmth are noted in both knees. The heart,
lung, abdominal, and neurologic examinations are normal.
Which of the following is the most likely diagnosis?
A. Behcet syndrome
B. Crohn disease
C. Sarcoidosis
D. Systemic lupus erythematous
E. Spondyloarthritis
Q31: a 55 y/o woman is evaluated for an 18 months history of increasingly sever knee pain with
the inability to arise when seated on the floor. She does not have pain at rest or nocturnal pain.
Medication are celecoxib and omeprazole. O/E vital signs are normal. Knee exam reveals
bilateral joint line tenderness with no warmth, erythema, or swelling. Plain AP knee radiographs
show mild medial joint space narrowing osteophytes; there are no erosions or osteopenia.
Which of the following is the most appropriate management?
A. Glucosamine supplements
B. Knee replacement
C. Physical therapy
D. Prednisone
E. Change celecoxib to ibuprofen
32. what is your most likely diagnosis?
A. lunate dislocation
C. Scaphoid fracture
D. lunate fracture
33. A 34-year old woman with a history of depression presents to the emergency department with
tachycardia, drowsiness and shallow breathing. After her initial presentation, she begins to
become confused and has a seizure. An ECG is performed and shows QT prolongation and
ventricular arrhythmia. An arterial blood gas shows metabolic acidosis. An overdose of which of
the following drugs is the most likely cause of patient symptoms?
A. Acetaminophen
B. Amitriptyline
C. Diazepam
D. digoxin
E. Methanol
34. A 52 year old man comes to ED because he has had vomiting, nausea, and abdominal pain
for the past 12 hours. He says he attempted suicide 3 days ago by “taking everything in the
medicine cabinet”. He was stuporous for approximately 12 hours after the overdose but felt
better the following day. At this time, he has jaundice and pain in the right upper quadrant.
Overdose of which of the following drugs is most likely to have caused the pain, vomiting, and
jaundice in this patient?
A. Acetaminophen
B. Aspirin
C. Cimetidine
D. Diphenhydramine
E. Triazolam
35. A 68 year old man with congestive heart failure presents to the ED with dehydration. He has
been vomiting and had diarrhea for the past 36 hours. His blood pressure is 105/75 mm Hg with
a pulse of 62 beats/minute. His blood digoxin level is 4.1 ng/ml (normal 1.0 to 2.6). which of the
following is most effective in treatment of the patient condition?
A. Deferoxamine
D. protamine sulphate
E. Sodium bicarbonate
36. no history of trauma, based on CT brain findings what is your next step?
A. Repeat CT brain after 6 hours
B. MRI brain
D. CTA (angiography)
E. Conventional angiography
37. A 50 year old man has brain tumor with increased intracranial pressure. Clinical examination
reveals convergent squint of the right eye. The patient is unable to rotate it externally. Which of
the following nerves is the single most likely affected?
A. Abducent
B. Maxillary
C. Oculomotor
D. Ophthalmic
E. Trochlear
38. A 30 year old man has bleeding from the nose. On examination; the bleeding comes from the
anterior inferior area of the septum. Which of the following arteries is the major contribution of
the blood in this area?
A. anterior ethmoidal
B. greater palatine
C. posterior ethmoidal
D. sphenopalatine
E. superior labial
39. A 35 year old woman has epigastric pain due to peptic ulcer for years. He suddenly feels
sever pain in the epigastrium. Examination reveals rigidity in the epigastrium (board-like
rigidity). Which of the following structures is the single most likely penetrated by the above
ulcer?
A. greater sac
B. left kidney
C. liver
D. pancreas
E. spleen
40. A 1 year old female has a palpable mass within one of her labia majora. Radiographic
examination reveals that a loop of intestine has herniated.
Which is the single most likely the diagnosis?
A. Direct inguinal hernia
C. Femoral hernia
E. Lumbar hernia
41. A 60 year old woman was found on routine screening to have iron deficiency anemia. Her
other labs parameter and physical examination was unremarkable. Which one of the following is
the most appropriate management of the patient?
A. prescribe an iron supplement
E. prescribe vitamin C
42. A 65 year old male with histologically confirmed adenocarcinoma of the colon presented to
the ER with per rectal bleeding, upon arrival he was hypotensive 85/60 and tachycardic. What is
your initial response in the emergency room?
A. staging CT abdomen and thorax to determine the TNM status
B. start chemotherapy as soon as possible
43. A 67 year old British man is brought to the ED in a stable condition with a history of
haematochezia. He has no relevant previous medical or drug history, works as a builder, is a non-
smoker and drinks 60 units of alcohol per week. What is the most likely cause of gastrointestinal
bleeding?
A. variceal bleeding is the most likely cause.
E. his predicted mortality attributable to this illness is higher than that of an inpatient who
develops UGIB.
44. A 72 year old man has had a sensation of retrosternal burning that is worse at night and after
meals for the last 4 months. It partially responds to ‘over the counter’ antacid treatment. Which is
the single most appropriate initial diagnostic investigation?
A. barium meal
B. barium swallow
C. CT scan thorax
D. EGD
E. 24h PH studies
45. 30 year old restrained driver was involved in a motor-vehicle crash. He is hemodynamically
stable and has a large seat belt sign on the abdomen. His abdomen is tender to palpation. In this
patient one should be most concerned about:
A. liver and spleen injury
C. pelvic fracture
D. hollow-viscus injuries
46. 32 year old female falls from the 10th floor of her apartment building in an apparent suicide
attempt. Upon presentation, the patient has obvious head and extremity injuries. Primary survey
reveals that the patient totally apnic. By which method is the immediate need for a definitive
airway in this patient best provided?
A. orotracheal intubation
B. nasotracheal intubation
C. percutaneous cricothyroidotomy
E. LMA
47. 45 years old engineer had sudden onset of severe abdominal pain, he smokes one peck per
day, on examination he has normal vital signs except for tachycardia (pulse 100 beat/minute), he
has tenderness on superficial palpation. Of the following, what is the most likely test to aid the
diagnosis of this patient?
A. flat plate of the abdomen
B. serum amylase
C. abdominal series
E. EGD
48. 29 years old female 2 weeks postpartum has right upper quadrant pain and vomiting, on
examination she has RUQ tenderness with +ve murphy’s sign. Which of the following abdominal
ultrasound finding confirms your diagnosis?
A. hepatomegaly
C. large stone in the common bile duct with dilated common bile duct
B. haemophilus influenza
C. staphylococcus aureus
D. streptococcus pyogenes
E. staphylococcus albus
50. A 42 year old obese diabetic man presents with 5*5 cm painful dark res colored swelling in
the upper back with multiple openings discharging pus.
A. an infection of the hair follicles
B. an insect bite
C. severe sunburn
E. spreading cellulitis
51. 19 year old man presents to ER after motor vehicle crash (MVC). Patient is disoriented and
complaining of severe chest pain in the right side. During primary survey, his O2 saturation is
91% on room air, no breath sound on the left with subcutaneous emphysema. Heart rate is 120
beat/min, but blood pressure in 80/60. What is the most likely cause of this hemodynamic status?
A. tension pneumothorax
B. open pneumothorax
C. simple pneumothorax
D. massive pneumothorax
E. occult pneumothorax
52. 33 year old female, came to ER after she fell down the stairs. She is unconscious,
hypotensive and hypoxic. What would be the best initial intervention to improve her oxygen
saturation?
A. high flow nasal cannula
B. face mask
D. endotracheal intubation
E. emergency tracheostomy
53. A 23 year old male patient with crohn’s disease develops an enterocutaneous fistula.
The metabolic abnormality likely to occur is:
A. hypocalcemia
B. hypochloremia
C. hypokalemia
D. hypophosphatasemia
E. hyperamylasemia
54. you are called by the on-call senior surgical nurse to assess a 74 year old male patient who
has undergone a right hemicolectomy earlier that same day. You are informed his pulse is 115
BPM and BP is 90/55. On arrival to the surgical ward, your immediate management should
consist of:
A. request 2 units of blood for transfusion and call a senior member of the surgical team.
B. arrange for a septic screen and an abdominal and pelvic CT scan with IV and oral
contrast.
C. complete the patient’s vitals and clinical assessment and commence resuscitations
measures as appropriate.
D. consent the patient for surgical re-exploration and arrange with operating room staff
immediate transfer.
E. review the patient’s medical records and obtain a full history from the patient relative.
55. A 71 year old diabetic man presents to the emergency room complaining of scrotal pain. On
examination, he had a small area of purplish denuded skin, with surrounding erythema and
tenderness. A CT scan shows some air within the scrotum and perineum. BP 90/50mmHg. HR
110/min, RR 24/min, T 39.6 C.
Test Result Normal values
Hb 130 142-176 g/L
INR 1.0 0.7-1.2 s
WBC 17 4-10.5 *109 /L
Platelet count 450 140-400 *109 /L
ReGculocyte count 0.6 0.5%-2.5%
APTT 28 22-37 s
B. IV immunoglobulin
C. Surgical debridement
56. A 63 year - old diabetic man is brought to the emergency room in severe discomfort and pain.
He was last able to urinate 14 hours ago. On examination, he has a suprapubic tender, globular
(spherical) mass. BP 140 / 90mmHg, HR 96 / min, RR 22 / min, T 36,6 ° C. Which of the
following is the most appropriate next step?
A) Urinalysis
B) Alpha blockers
C) Oral antibiotics
D. Abdominal ultrasound
E. Foley catheter insertion
57.68 years old male patient presented to the OPD clinic with cramping left thigh pain and calf
for 8 months. The pain increases with walking and relived by rest for 15 min. He came to the
OPD because the pain has been more frequent and comes at rest. The patient Has history of DM,
HTN, Hypercholesteremia, and he quit smoking 5 years ago. In your evaluation all of the
following findings are expected EXCEPT: a. Absent left posterior tibial pulse
b. Aortic and lower limbs CT angiography showed 90% Left external iliac artery occlusion.
c. Left foot burger angle of 10 degrees with hyperemic response
d. Left ankle / brachial index of 0.9
e. Left big toe capillary refill time of 45 seconds
58.70 years old female presented to the OPD with history of right calf intermittent claudication
for six months. Claudication distance has been stable at 300 meters and relieved at rest. She is
known diabetic and hypertensive with Hb1c of 11%. Patient had Lower limbs CT angiography
which showed 80% right common femoral artery occlusion and 70% right popliteal artery
occlusion with antegrade filling of both anterior and posterior tibial arteries. Her best option for
treatment at this stage would be:
a. Right common femoral artery endarterectomy
b. Right femoral popliteal artery bypass using reversed saphenous venous graft
c. Optimize medical treatment (control BP and DM and start on ASA and Pentoxifylline)
d. Balloon angioplasty of the right common femoral artery
e. Reassurance and follow up within three months
59.A 2.8 kg. neonate with excessive salivation develops respiratory distress. Attempts to pass an
orogastric catheter fail because the catheter coils in the back of the throat. A chest film is
obtained and shows right upper lobe atelectasis and a normal gas pattern in abdomen. The most
likely diagnosis is:
a) Congenital diaphragmatic hernia.
b) Proximal esophageal atresia with a distal tracheoesophageal (TE) fistula.
C) Pyloric stenosis.
d) Duodenal atresia.
e) Inguinal hernia
61. A 65 male with history of HTN and DM seen in the clinic with new diagnosis of a - fib.
Hemodynamically stable. All investigations normal. Which of the following is the next step?
A. Aspirin + metoprolol
B Warfarin + metoprolol
C. Cardioversion
D. Amiodarone
E. Cardiology referral
62.A 25 - year - old man presented with 2 weeks history of on and off fever associated with sore
throat and fatigability. His vitals are normal apart from T 39 C. His blood work revealed the
following: WBC 89000 / microL Potassium HGB 6 g / dl Uric acid Plts 10,000 / microL
Creatinine 16.5mEq / L 12 mg / dL 2,5 mg / dl Which of the following is the single most likely
diagnosis?
a. Acute kidney injury.
b. Acute gout
c. Leukemoid reaction
d. Spontaneous tumor lysis syndrome
e. Thrombotic thrombocytopenic purpura
63. A 20 - year - old woman has 48 - hour history of progressive headache and fever that are
associated with nausea, vomiting, and epigastric pain. On examination, she appears restless and
confused. Temp 38.1 C, pulse 110 bpm, BP 150 / 90mmHg, and RR 22 / min. remainder of
physical examination is unremarkable. Laboratory studies show Hb 9.7g / dl, MCV 102 FL,
WBC 6 X10 ° / L, PLT 5 X109 / L, haptoglobin 10mg / dl, LDH 6000 / l, and creatinine 1. 4mg /
dl. A peripheral blood film shows fragmented RBCs. Direct Anti - globulin test is negative.
Which of the following is the most likely diagnosis?
a. Aplastic anemia
b. Disseminated intravascular coagulation,
c. Thrombotic thrombocytopenic purpura,
d. Autoimmune hemolytic anemia with immune thrombocytopenic purpura
e.Heparin Induced Thrombocytopenia
64.A 35 - year old male, presented to emergency room with fever and cough. He was well until
3 days earlier, when he suffered the onset of nasal stuffiness, mild sore throat, and a cough
productive of small amounts of clear sputum. Patient smokes one pack of cigarettes daily.
Today, he decided to seek physician assistance because of an increase in temperature to 38.3 ° C
and worsening cough. Chest X - ray showed right lower lobe consolidation. What is the most
likely diagnosis?
A. Acute bronchitis
B. Community acquired pneumonia
C. hospital acquired pneumonia
D. Pulmonary tuberculosis
E. Interstitial pneumonitis
65.50 year old male patient hospitalized with a diagnosis of community acquired pneumonia.
What is the most effective therapy for this patient?
A. Intravenous Methylprednisolone
B. Inhaled Salbutamol
C. Inhaled Budesonide
D. Intravenous Ceftriaxone and Azithromycin
E. Oral Dextromethorphan (cough suppressant)
66.30 year old male known case of bronchial asthma & insulin dependent DM presenting to ER
department because of progressive shortness of breath of ER department. He had mild dry cough
but no wheeze. In addition he had frequent loose bowel motions without abdominal pain or fever
The bowel motions didn't contain blood nor had black color.on examination BR 90/60 HR 110
Temp 36 RR 26 Oxygen saturation 95 % on room air. JVP 1 cm above sternal angle Chest and
cardiac examination was normal. His lab work showed: Na 120 Chloride 95 HCO3 15 PH 7.25
PO2 100 PCO2 30. The most appropriate management is:
a) Inhaled bronchodilator & Intravenous steroids
b) Intravenous Insulin
c) Sputum & stool cultures
d. Intravenous fluid bolus & maintenance
e. Chest & Abdominal X rays
67. A 57 - year - old man has right side weakness. His blood pressure is 220/140. Funduscopic
exam decrease in power and sensation in upper and lower limbs. His past medical history is
significant demonstrates arteriolar narrowing and arteriovenous nicking. Neurological exam
revealed for hyperglycemia, hypertension, and gout. His medications are losartan 50 mg daily.
medication allergies. His labs within normal limits. Which of the following is an appropriate
treatment option?
A. Nitroglycerin IV
B. Amlodipine orally
C. Sodium nitroprusside IV
D. Nimodipine orally
E. Labetalol IV
68. A 49 - year - old man with history of hypertension presents to the emergency department with
acute onset of right hemiparesis and aphasia. The time he was last seen normal was about 45
minutes prior to arrival. The National Institutes of Health Stroke Scale (NIHSS) score is 14.
Which of the following is the best next step?
a. Start intravenous tissue plasminogen activator (tPA)
b. Get a brain CT scan
c. Give aspirin 325 mg once
d. Start intravenous heparin
e. Get a brain MRI
69. A 49 - year - old woman presents with acute onset of left hemiplegia and right - side facial
weakness, involving the upper and lower facial movements. Which of the following is the most
likely diagnosis?
a. Right pontine infarct
b. Left pontine infarct
c. Right midbrain infarct
d. Left midbrain infarct
e. Right MCA infarct
70.a 70 year old man admitted with right frontal intracerebral hemorrhage. On day 3 he
developed generalized seizure that aborted after 10 minutes by two doses of intravenous
benzodiazepines. What is the most appropriate next step in managements?
a) Intravenous valproic acid loading followed by maintenance
b) Oral carbamazepine
c) Start antiepiletic if second seizure developed
d) Start midazolam infusion
e) Start oral antiepileptic if EEG showed an epileptic discharges
71. A 33 - year - old male with history of type 1 diabetes who was brought to emergency room
with a history of altered mental status and found to have diabetes ketoacidosis. His PH is 6.8.
Which of the following statements is correct regarding the use of sodium bicarbonate in DKA?
A. Sodium bicarbonate can be added if PH <6.9
B. Sodium bicarbonate improves mortality in diabetes ketoacidosis
C. The use of Sodium bicarbonate is limited to type 1 diabetes patients
D. Sodium bicarbonate is associated with post treatment metabolic acidosis
E. Sodium bicarbonate can lead to severe hyperkalemia
72. A 60 year old man, who has COPD with frequent exacerbations and glucocorticoid treatment,
presents to ER with 1 - day history of increasing fatigue, nausea, and vomiting. On
examination, he is afebrile, HR 120 bpm, BP 97 / 75mmHg, and O2 saturation 92% on room air.
He has dry mucous membranes and diffusely tender abdomen without guarding or rebound
tenderness. His investigations reveal hyponatremia and evidence of UTI on urine analysis.
Which of the following is the most appropriate diagnostic test?
a) Random serum ACTH and cortisol
b) 24 - hour urine free cortisol
c) Late night salivary cortisol
d) ACTH stimulation test
73. A 42 - year - old male with a history of alcoholism presents to the hospital with confusion,
diplopia (ophthalmoparesis), unsteady gait, and nystagmus. What is the most likely cause of
these symptoms?
A. Thiamine deficiency
B. Hypoglycemia
C. Stroke
D. Hepatic encephalopathy
E. Hypothyroidism
74.A 53 - year - old woman with no past medical history, comes to the emergency department
with 4 day history of palpitation. She also complains of fatigue and dyspnea. Her BP is 153/72
and HR is 137 bpm, O2 sat was 95% on room air. On physical examination shows an irregularly
irregular rhythm. Auscultation reveals no crackles. Which of the following is the best next step
in management of this patient?
a) Adenosine
b) Metoprolol
C) Amiodarone
d) Heparin
e) Synchronized cardioversion
76.A 65 year old woman has a longstanding history of primary hypothyroidism with poor
compliance to treatment. She presents to ER with decreased level of consciousness and you are
concerned of her having myxedema coma. Which of the following abnormality will support
your working diagnosis?
a) Hypernatremia
b) Hypercapnia
c) Hypercalcemia
d) Hyperthermia
77.50 year old male presents with chest pain that occurred 2 hours ago, ECG shows ST elevation
V1-V4.
Which of the following are contraindications to thrombolysis?
A. Creatinine> 2.8
B. Systolic BP> 200
C. Absence of chest pain at present time
D. Gastrectomy for GI bleed 6 months ago
E. No contraindication
78. A 54 year old man has a 3 hour history of chest pain and dyspnea. He is 1 - week post
colectomy for colon cancer. On examination, Temp 37.5ºC, BP 110/60 mmHg, HR 130 / min,
and RR 24 / min. Oxygen saturation is 88% on room air and 94% on oxygen, 4 L / min. Cardiac
examination shows tachycardia. Breath sounds are normal. Serum creatinine concentration is 3
mg / dl (normal 0.84 1.2 mg / dl) and his complete blood count and coagulation profile is normal.
CT pulmonary angiography shows left sided pulmonary embolism. Which of the following is
the most appropriate immediate treatment?
A. Unfractionated heparin
B. Rivaroxiban
C. Enoxaparin
D. Fondaparinux
E. Dabigatran
79. A 20 year old male, with a medical history significant for active intravenous drug user
presented to emergency room complaining of persistent fever. Pertinent positive on physical
examination are temperature of 39.3 ° C, multiple pink, painful nodules on the palmar surface of
both hands and grade 3 pansystolic murmur on fourth left parasternal area. Three sets of blood
culture are sent What is the most likely cause of persistent fever?
A. Infective endocarditis
B. Lung abscess
C. Acute rheumatic fever
D. Drug induced fever
E. Allergic reaction
80. You were called to manage a 35-year male motor vehicle accident victim who has sustained
multiple leg fractures in addition to splenic and liver lacerations few days ago
The patient developed sudden shortness of breath and central chest pain worsening with deep
breathing No associated cough, wheeze or fever. His legs had been in casts from mid tibia till
above knee. BP 90/60) HR 130 regular, Temp 37, RR32, Oxygen Saturation 88% on room air.
Chest examination revealed central trachea, symmetrical air entry without added sounds. CVS
exam showed elevated JVP with loud pulmonary component of second sound. Leg exams
showed diameter difference at mid thigh showing larger left leg. Laboratory work up showed
drop in hemoglobin from 10 to 8 and stable WBC of 12000 in addition to normal cardiac
enzymes and chest X ray. The most appropriate next step is:
a) D dimer testing
b) Doppler US both legs
c) Echocardiogram
d) Start anticoagulant empirically
e) Transfuse packed red blood cells
6th year Summative exam - 1st term
Q11: a 65 y.o male with hearing loss. O/E he has ipsilateral secretary otitis media and painless
neck mass. What is his most likely diagnosis?
A. AIDS
B. Parapharyngial tumer
C. Glandular fever
D. Nasopharyngeal carcinoma
O
E. Hypopharngeal carcinoma
OB. FESS
C. Biopsy
D. Observation a
Q13: a 20 y/o young man had long standing foul smelling ear discharge, recently he has
developed otalgia, fever, headache and vomiting. What is his likely diagnosis?
A. Brain abscess
B. Bezolds abscess
C. Perilabyrinthine fistula
D. Otitis externa
E. Meningitis
e
Q14: a 50 y/o male pt. Presented with otalgia. O/E both external auditory canal and tympanic
membrane are normal. What is the most likely site of origin of his pain?
A. Temporomandibular joint
B. Subglottic region
C. Nasopharynx
D. Eye
E. Maxilla
Q16: a 40 y/o female with slowly progressive conductive deafness with normal drum & ET
function. Her CHL worsened after her last pregnancy. What is the most likely diagnosis?
A. OME
B. Otosclerosis
C. Malingering
D. Tympanosclerosis
E. Ossicular discontinuity
Q17: 60 y/o poorly controlled diabetic patient presents to clinic with otalgia purulent ear
discharge for the last 3 weeks. O/E the right ear shows mucopurulent discharge and granulation.
What is the most likely cranial nerve to be affected in this condition?
A. Trigeminal
B. Facial
C. Glossopharyngeal
D. Vagus
E. Optic
Q18: 55 y/o male pt. Presented to OPD with right shoulder pain for over 3 months, x-ray right
shoulder was normal and you are suspecting rotator cuff tendonitis. He was treated
conservatively with NSAID. 3 months lated at his follow up appointment you noticed weakness
in shoulder abduction and rotation. What is the test of choice to rule out rotator cuff tear?
A. Shoulder CT
B. Shoulder MRI
C. Shoulder US
D. Shoulder x-ray with contrast
E. Shoulder x-ray in 3 views
Q19: 30 y/o lady involved in MVA, she sustained tibia fx and has 2 cm open laceration over the
fx site, it’s clean and no contamination. What will be the treatment of choice?
A. Close reduction and above knee cast
B. Close reduction and below knee cast
C. Irrigation and debridement, close reduction and intramedullary nail
D. Irrigation and debridement, open reduction with plate and screws
E. Open reduction with external fixation & debridement
Q20: 25 y/o basketball player, sustained left shoulder dislocation at the field while playing, you
order x-ray and confirmed anterior shoulder dislocation. The preferred method of treatment is:
A. Open reduction under GA in OR
B. Close reduction under GA in OR
C. Close reduction under conscious sedation in ER
D. Close reduction under conscious sedation in ER with vital signs monitoring
E. Open reduction under regional anesthesia
Q21: 70 y/o female pt. Had a fall at home and sustained displaced femoral neck fx. The best
treatment choice is:
A. Bed rest and traction till fx heals then mobilization
B. Close reduction and dynamic hip screw
C. Open reduction and dynamic hip screw
D. Hip hemi arthroplasty
E. Skeletal traction as she is a high risk patient
Q22: in grade 3A comminuted mid tibia fx, the soft tissue defect over the mid shaft tibia and
after fixation with intramedullary nail you were unable to approximate the wound edges and
defect was over 2 cm over the tibia bone . What choice of treatment that is acceptable:
A. Leave wound open, multiple irrigation and debridement then perform skin graft
B. Immediate skin graft after nail fixation
C. Insult plastic surgeon prior fixation to consider local flap with skin graft
D. Close wound and defect with tension suture
E. Primary closure using skin metallic clips
Q23: what is the next step in treating patient with right shoulder pain (subacromion
impengement) failed full course of NSAID and intense physiotherapy program, shoulder MRI
shows tendon inflammation with no sign of acute cuff…..
A. Give additional NSAID on top o what pt. Is taking
B. Corticosteroid subacromion injection
C. Corticosteroid shoulder injection
D. Corticosteroid acromion clavicular AC joint injection
E. Surgical intervention
Q24: The best treatment choice of distal radius fx in 65 y/o male, minimal displacement on x-ray
is:
A. Open reduction and K-wire fixation in comminutedEx
B. Close reduction and above elbow cast application below
C. Close reduction and K-wire fixation
D. Application of mini wrist external fixation
E. NSAID & physiotherapy
Q25: a 72y/o man presented with an acutely painful right knee. O/E he has a T=37c with a hot,
swollen right knee. Of relevance amongst his investigations, was his WBC=12. 6*109/L and knee
x-ray revealed reduced joint space and calcification of the articular cartilage. Culture of aspirated
fluid revealed no growth . What is the most likely diagnosis?
A. Gout
O
B. Pseudogout
C. RA L
D. Prosaic monoarthropathy L
E. Septic arthritis
K
Q26: A 25-year-old man is evaluated for the gradual onset of bilateral low back pain without
radiation to the lower extremities daily, with increasing severity over the past year. The pain now
awakens him during the night 2 to 3 times per week, with morning stiffness lasting more than an
hour. He has improvement with exercise and no improvement at rest. He takes ibuprofen with
some improvement.
On physical examination, vital signs are normal. Limited lateral bending bilaterally and a
reduction in forward flexion at the lumbar spine are noted. The remainder of the examination is
normal.
An anteroposterior plain radiograph of the pelvis and sacroiliac joints is unremarkable.
Which of the following is the most appropriate diagnostic test to perform next?
A. Bone scan
B. CT of the lumbar spine
eC. MRI of the sacroiliac joints
D. Radiography of the hip joints
E. All of the above
Q27: a 62 y/o woman presents complaining of hand pain bilaterally that has been gradually
progressive over the past 2 years. She has previously worked in a factory making gloves for
more than 35 years. You suspect osteoarthritis. All of the following factors on history or physical
ne
examination are characteristic of his diagnosis EXCEPT:
A. Evidence of bilateral swelling and warmth affecting the wrists only
e
B. Joint space narrowing and osteophytes at he proximal and distal interphalangeal joints on
x-ray
C. Pain that becomes worse when preparing meals
D. Presence of Hebeden’s nodes
E. Stiffness that lasts for a few minutes after brief periods of rest with occasional locking of
the more affecting joints
Q28: a 33y/o woman is evaluated for concerns about the development of RA. She has some mild
joint discomfort in the end of the day. She has been smoking five cigarettes per day for the past 6
years. Her identical twin sister was recently diagnosed with seropositive RA. Her physical
examination is unremarkable. Which of the following is the most appropriate preventive measure
for this patient at risk for developing RA?
A. Avoid food/ drinks containing high fructose syrup
B. Begin hydroxychloroquine
C. Begin probiotic
O D. Smoking cessation
E. Modification of work
Q29: a 27y/o woman is evaluated for a sudden onset of joint pain and significant stiffness in her
fingers, wrists, knees, and ankles for the past 5 days. She had a brief fever with muscle aches a
few days ago and a faint pink rash on her arms and legs at the same time. She reports no other
symptoms. She works at daycare. O/E there is mild warmth and tenderness of the first through
fifth proximal interphalangeal and metacarpophalangeal joints bilaterally as well as wrists.
Which of the following will most likely confirm the diagnosis?
A. Antinuclear antibodies
B. HIV testing
C. Parvovirus B19 testing
a
D. RF
E. Anti-CCP antibodies
Q30: A 31-year-old man is evaluated In the emergency department for fever and red painful eyes.
He reports a 1-month history of intermittent painful oral and genital ulcers, knee pain, and fever
and 1 week of bilateral eye redness, pain and photophobia.
On physical examination, temperature is 38.3 and pulse rate is 100/min. Slit lamp examination
reveals white cells in the anterior chambers of both eyes. There are aphthous ulcers on the tongue
and one aphthous ulcer on the scrotum. Swelling and warmth are noted in both knees. The heart,
lung, abdominal, and neurologic examinations are normal.
Which of the following is the most likely diagnosis?
O
A. Behcet syndrome
B. Crohn disease
C. Sarcoidosis
D. Systemic lupus erythematous
E. Spondyloarthritis
Q31: a 55 y/o woman is evaluated for an 18 months history of increasingly sever knee pain with
the inability to arise when seated on the floor. She does not have pain at rest or nocturnal pain.
Medication are celecoxib and omeprazole. O/E vital signs are normal. Knee exam reveals
bilateral joint line tenderness with no warmth, erythema, or swelling. Plain AP knee radiographs
show mild medial joint space narrowing osteophytes; there are no erosions or osteopenia.
Which of the following is the most appropriate management?
A. Glucosamine supplements
B. Knee replacement
y
C. Physical therapy
D. Prednisone oral
E. Change celecoxib to ibuprofen same
32. what is your most likely diagnosis?
C. Scaphoid fracture
I
D. lunate fracture
33. A 34-year old woman with a history of depression presents to the emergency department with
tachycardia, drowsiness and shallow breathing. After her initial presentation, she begins to
become confused and has a seizure. An ECG is performed and shows QT prolongation and
ventricular arrhythmia. An arterial blood gas shows metabolic acidosis. An overdose of which of
the following drugs is the most likely cause of patient symptoms?
A. Acetaminophen
B. Amitriptyline
C. Diazepam
D. digoxin
E. Methanol
34. A 52 year old man comes to ED because he has had vomiting, nausea, and abdominal pain
for the past 12 hours. He says he attempted suicide 3 days ago by “taking everything in the
medicine cabinet”. He was stuporous for approximately 12 hours after the overdose but felt
better the following day. At this time, he has jaundice and pain in the right upper quadrant.
Overdose of which of the following drugs is most likely to have caused the pain, vomiting, and
jaundice in this patient?
A. Acetaminophen
B. Aspirin
C. Cimetidine
D. Diphenhydramine
E. Triazolam
35. A 68 year old man with congestive heart failure presents to the ED with dehydration. He has
been vomiting and had diarrhea for the past 36 hours. His blood pressure is 105/75 mm Hg with
a pulse of 62 beats/minute. His blood digoxin level is 4.1 ng/ml (normal 1.0 to 2.6). which of the
following is most effective in treatment of the patient condition?
A. Deferoxamine
D. protamine sulphate
E. Sodium bicarbonate
36. no history of trauma, based on CT brain findings what is your next step?
A. Repeat CT brain after 6 hours SAH
B. MRI brain
D. CTA (angiography)
look foraneurysm
E. Conventional angiography
37. A 50 year old man has brain tumor with increased intracranial pressure. Clinical examination
reveals convergent squint of the right eye. The patient is unable to rotate it externally. Which of
the following nerves is the single most likely affected?
A. Abducent
O
B. Maxillary
C. Oculomotor
D. Ophthalmic
E. Trochlear
38. A 30 year old man has bleeding from the nose. On examination; the bleeding comes from the
anterior inferior area of the septum. Which of the following arteries is the major contribution of
the blood in this area?
A. anterior ethmoidal
B. greater palatine
C. posterior ethmoidal
D. sphenopalatine
B. left kidney
C. liver
D. pancreas
E. spleen
40. A 1 year old female has a palpable mass within one of her labia majora. Radiographic
examination reveals that a loop of intestine has herniated.
Which is the single most likely the diagnosis?
A. Direct inguinal hernia
C. Femoral hernia
O
D. Indirect inguinal hernia
E. Lumbar hernia
41. A 60 year old woman was found on routine screening to have iron deficiency anemia. Her
other labs parameter and physical examination was unremarkable. Which one of the following is
the most appropriate management of the patient?
A. prescribe an iron supplement
E. prescribe vitamin C
42. A 65 year old male with histologically confirmed adenocarcinoma of the colon presented to
the ER with per rectal bleeding, upon arrival he was hypotensive 85/60 and tachycardic. What is
your initial response in the emergency room?
A. staging CT abdomen and thorax to determine the TNM status
B. start chemotherapy as soon as possible
e
C. canula insertion for IV fluids and cross match
43. A 67 year old British man is brought to the ED in a stable condition with a history of
haematochezia. He has no relevant previous medical or drug history, works as a builder, is a non-
smoker and drinks 60 units of alcohol per week. What is the most likely cause of gastrointestinal
bleeding?
A. variceal bleeding is the most likely cause.
eD. the bleeding will have originated distal to the ligament of treitz.
E. his predicted mortality attributable to this illness is higher than that of an inpatient who
develops UGIB.
44. A 72 year old man has had a sensation of retrosternal burning that is worse at night and after
meals for the last 4 months. It partially responds to ‘over the counter’ antacid treatment. Which is
the single most appropriate initial diagnostic investigation?
A. barium meal -
B. barium swallow
C. CT scan thorax
DD. EGD
E. 24h PH studies
45. 30 year old restrained driver was involved in a motor-vehicle crash. He is hemodynamically
stable and has a large seat belt sign on the abdomen. His abdomen is tender to palpation. In this
patient one should be most concerned about:
C. pelvic fracture
D. hollow-viscus injuries
46. 32 year old female falls from the 10th floor of her apartment building in an apparent suicide
attempt. Upon presentation, the patient has obvious head and extremity injuries. Primary survey
reveals that the patient totally apnic. By which method is the immediate need for a definitive
airway in this patient best provided?
A. orotracheal intubation
B. nasotracheal intubation
C. percutaneous cricothyroidotomy
E. LMA
47. 45 years old engineer had sudden onset of severe abdominal pain, he smokes one peck per
day, on examination he has normal vital signs except for tachycardia (pulse 100 beat/minute), he
has tenderness on superficial palpation. Of the following, what is the most likely test to aid the
diagnosis of this patient?
A. flat plate of the abdomen
B. serum amylase
C. abdominal series
E. EGD
48. 29 years old female 2 weeks postpartum has right upper quadrant pain and vomiting, on
examination she has RUQ tenderness with +ve murphy’s sign. Which of the following abdominal
ultrasound finding confirms your diagnosis?
A. hepatomegaly
C. large stone in the common bile duct with dilated common bile duct
B. haemophilus influenza
C. staphylococcus aureus
50. A 42 year old obese diabetic man presents with 5*5 cm painful dark res colored swelling in
the upper back with multiple openings discharging pus.
A. an infection of the hair follicles
B. an insect bite
C. severe sunburn
E. spreading cellulitis
51. 19 year old man presents to ER after motor vehicle crash (MVC). Patient is disoriented and
complaining of severe chest pain in the right side. During primary survey, his O2 saturation is
91% on room air, no breath sound on the left with subcutaneous emphysema. Heart rate is 120
beat/min, but blood pressure in 80/60. What is the most likely cause of this hemodynamic status?
A. tension pneumothorax
D. massive pneumothorax
E. occult pneumothorax
52. 33 year old female, came to ER after she fell down the stairs. She is unconscious,
hypotensive and hypoxic. What would be the best initial intervention to improve her oxygen
saturation?
A. high flow nasal cannula
B. face mask
D. endotracheal intubation
E. emergency tracheostomy
53. A 23 year old male patient with crohn’s disease develops an enterocutaneous fistula.
The metabolic abnormality likely to occur is:
A. hypocalcemia
B. hypochloremia
C. hypokalemia
D. hypophosphatasemia
E. hyperamylasemia
54. you are called by the on-call senior surgical nurse to assess a 74 year old male patient who
has undergone a right hemicolectomy earlier that same day. You are informed his pulse is 115
BPM and BP is 90/55. On arrival to the surgical ward, your immediate management should
consist of:
A. request 2 units of blood for transfusion and call a senior member of the surgical team.
B. arrange for a septic screen and an abdominal and pelvic CT scan with IV and oral
contrast.
C. complete the patient’s vitals and clinical assessment and commence resuscitations
measures as appropriate.
D. consent the patient for surgical re-exploration and arrange with operating room staff
immediate transfer.
E. review the patient’s medical records and obtain a full history from the patient relative.
55. A 71 year old diabetic man presents to the emergency room complaining of scrotal pain. On
examination, he had a small area of purplish denuded skin, with surrounding erythema and
tenderness. A CT scan shows some air within the scrotum and perineum. BP 90/50mmHg. HR
110/min, RR 24/min, T 39.6 C.
Test Result Normal values
Hb 130 142-176 g/L
INR 1.0 0.7-1.2 s
WBC 17 4-10.5 *109 /L
Platelet count 450 140-400 *109 /L
ReGculocyte count 0.6 0.5%-2.5%
APTT 28 22-37 s
B. IV immunoglobulin
C. Surgical debridement
oD. Incision and drainage
E. Administer intravenous antibiotics
56. A 63 year - old diabetic man is brought to the emergency room in severe discomfort and pain.
He was last able to urinate 14 hours ago. On examination, he has a suprapubic tender, globular
(spherical) mass. BP 140 / 90mmHg, HR 96 / min, RR 22 / min, T 36,6 ° C. Which of the
following is the most appropriate next step?
A) Urinalysis
B) Alpha blockers
C) Oral antibiotics
D. Abdominal ultrasound
E. Foley catheter insertion
O
57.68 years old male patient presented to the OPD clinic with cramping left thigh pain and calf
for 8 months. The pain increases with walking and relived by rest for 15 min. He came to the
OPD because the pain has been more frequent and comes at rest. The patient Has history of DM,
HTN, Hypercholesteremia, and he quit smoking 5 years ago. In your evaluation all of the
following findings are expected EXCEPT: a. Absent left posterior tibial pulse
b. Aortic and lower limbs CT angiography showed 90% Left external iliac artery occlusion.
c. Left foot burger angle of 10 degrees with hyperemic response
e
d. Left ankle / brachial index of 0.9
e. Left big toe capillary refill time of 45 seconds
58.70 years old female presented to the OPD with history of right calf intermittent claudication
for six months. Claudication distance has been stable at 300 meters and relieved at rest. She is
known diabetic and hypertensive with Hb1c of 11%. Patient had Lower limbs CT angiography
which showed 80% right common femoral artery occlusion and 70% right popliteal artery
occlusion with antegrade filling of both anterior and posterior tibial arteries. Her best option for
treatment at this stage would be:
a. Right common femoral artery endarterectomy x
b. Right femoral popliteal artery bypass using reversed saphenous venous graft
c. Optimize medical treatment (control BP and DM and start on ASA and Pentoxifylline)
d. Balloon angioplasty of the right common femoral artery
e. Reassurance and follow up within three months
59.A 2.8 kg. neonate with excessive salivation develops respiratory distress. Attempts to pass an
orogastric catheter fail because the catheter coils in the back of the throat. A chest film is
obtained and shows right upper lobe atelectasis and a normal gas pattern in abdomen. The most
likely diagnosis is:
a) Congenital diaphragmatic hernia.
b) Proximal esophageal atresia with a distal tracheoesophageal (TE) fistula.
C) Pyloric stenosis.
d) Duodenal atresia.
e) Inguinal hernia
61. A 65 male with history of HTN and DM seen in the clinic with new diagnosis of a - fib.
Hemodynamically stable. All investigations normal. Which of the following is the next step?
A. Aspirin + metoprolol
O
B Warfarin + metoprolol
C. Cardioversion
D. Amiodarone
E. Cardiology referral
62.A 25 - year - old man presented with 2 weeks history of on and off fever associated with sore
throat and fatigability. His vitals are normal apart from T 39 C. His blood work revealed the
following: WBC 89000 / microL Potassium HGB 6 g / dl Uric acid Plts 10,000 / microL
Creatinine 16.5mEq / L 12 mg / dL 2,5 mg / dl Which of the following is the single most likely
diagnosis?
a. Acute kidney injury.
If
b. Acute gout r
c. Leukemoid reaction
d. Spontaneous tumor lysis syndrome
C
e. Thrombotic thrombocytopenic purpura
63. A 20 - year - old woman has 48 - hour history of progressive headache and fever that are
associated with nausea, vomiting, and epigastric pain. On examination, she appears restless and
confused. Temp 38.1 C, pulse 110 bpm, BP 150 / 90mmHg, and RR 22 / min. remainder of
physical examination is unremarkable. Laboratory studies show Hb 9.7g / dl, MCV 102 FL,
WBC 6 X10 ° / L, PLT 5 X109 / L, haptoglobin 10mg / dl, LDH 6000 / l, and creatinine 1. 4mg /
dl. A peripheral blood film shows fragmented RBCs. Direct Anti - globulin test is negative.
Which of the following is the most likely diagnosis?
a. Aplastic anemia
b. Disseminated intravascular coagulation,
c. Thrombotic thrombocytopenic purpura,
d. Autoimmune hemolytic anemia with immune thrombocytopenic purpura
e.Heparin Induced Thrombocytopenia
64.A 35 - year old male, presented to emergency room with fever and cough. He was well until
3 days earlier, when he suffered the onset of nasal stuffiness, mild sore throat, and a cough
productive of small amounts of clear sputum. Patient smokes one pack of cigarettes daily.
Today, he decided to seek physician assistance because of an increase in temperature to 38.3 ° C
and worsening cough. Chest X - ray showed right lower lobe consolidation. What is the most
likely diagnosis?
A. Acute bronchitis
B. Community acquired pneumonia
C. hospital acquired pneumonia
D. Pulmonary tuberculosis
E. Interstitial pneumonitis
65.50 year old male patient hospitalized with a diagnosis of community acquired pneumonia.
What is the most effective therapy for this patient?
A. Intravenous Methylprednisolone
B. Inhaled Salbutamol
C. Inhaled Budesonide
O
D. Intravenous Ceftriaxone and Azithromycin
E. Oral Dextromethorphan (cough suppressant)
66.30 year old male known case of bronchial asthma & insulin dependent DM presenting to ER
department because of progressive shortness of breath of ER department. He had mild dry cough
but no wheeze. In addition he had frequent loose bowel motions without abdominal pain or fever
The bowel motions didn't contain blood nor had black color.on examination BR 90/60 HR 110
Temp 36 RR 26 Oxygen saturation 95 % on room air. JVP 1 cm above sternal angle Chest and
cardiac examination was normal. His lab work showed: Na 120 Chloride 95 HCO3 15 PH 7.25
PO2 100 PCO2 30. The most appropriate management is:
a) Inhaled bronchodilator & Intravenous steroids
b) Intravenous Insulin
c) Sputum & stool cultures
d. Intravenous fluid bolus & maintenance
o
e. Chest & Abdominal X rays
67. A 57 - year - old man has right side weakness. His blood pressure is 220/140. Funduscopic
exam decrease in power and sensation in upper and lower limbs. His past medical history is
significant demonstrates arteriolar narrowing and arteriovenous nicking. Neurological exam
revealed for hyperglycemia, hypertension, and gout. His medications are losartan 50 mg daily.
medication allergies. His labs within normal limits. Which of the following is an appropriate
treatment option?
A. Nitroglycerin IV
B. Amlodipine orally
C. Sodium nitroprusside IV
D. Nimodipine orally
E. Labetalol IV
68. A 49 - year - old man with history of hypertension presents to the emergency department with
acute onset of right hemiparesis and aphasia. The time he was last seen normal was about 45
minutes prior to arrival. The National Institutes of Health Stroke Scale (NIHSS) score is 14.
Which of the following is the best next step?
a. Start intravenous tissue plasminogen activator (tPA)
b. Get a brain CT scan
c. Give aspirin 325 mg once
d. Start intravenous heparin
e. Get a brain MRI
69. A 49 - year - old woman presents with acute onset of left hemiplegia and right - side facial
weakness, involving the upper and lower facial movements. Which of the following is the most
likely diagnosis?
a. Right pontine infarct
b. Left pontine infarct
c. Right midbrain infarct
d. Left midbrain infarct
e. Right MCA infarct
70.a 70 year old man admitted with right frontal intracerebral hemorrhage. On day 3 he
developed generalized seizure that aborted after 10 minutes by two doses of intravenous
benzodiazepines. What is the most appropriate next step in managements?
a) Intravenous valproic acid loading followed by maintenance
b) Oral carbamazepine
c) Start antiepiletic if second seizure developed
d) Start midazolam infusion
e) Start oral antiepileptic if EEG showed an epileptic discharges
71. A 33 - year - old male with history of type 1 diabetes who was brought to emergency room
with a history of altered mental status and found to have diabetes ketoacidosis. His PH is 6.8.
Which of the following statements is correct regarding the use of sodium bicarbonate in DKA?
A. Sodium bicarbonate can be added if PH <6.9
B. Sodium bicarbonate improves mortality in diabetes ketoacidosis
C. The use of Sodium bicarbonate is limited to type 1 diabetes patients
D. Sodium bicarbonate is associated with post treatment metabolic acidosis
E. Sodium bicarbonate can lead to severe hyperkalemia
72. A 60 year old man, who has COPD with frequent exacerbations and glucocorticoid treatment,
presents to ER with 1 - day history of increasing fatigue, nausea, and vomiting. On
examination, he is afebrile, HR 120 bpm, BP 97 / 75mmHg, and O2 saturation 92% on room air.
He has dry mucous membranes and diffusely tender abdomen without guarding or rebound
tenderness. His investigations reveal hyponatremia and evidence of UTI on urine analysis.
Which of the following is the most appropriate diagnostic test?
a) Random serum ACTH and cortisol
b) 24 - hour urine free cortisol
c) Late night salivary cortisol
d) ACTH stimulation test
73. A 42 - year - old male with a history of alcoholism presents to the hospital with confusion,
diplopia (ophthalmoparesis), unsteady gait, and nystagmus. What is the most likely cause of
these symptoms?
A. Thiamine deficiency
B. Hypoglycemia
C. Stroke
D. Hepatic encephalopathy
E. Hypothyroidism
74.A 53 - year - old woman with no past medical history, comes to the emergency department
with 4 day history of palpitation. She also complains of fatigue and dyspnea. Her BP is 153/72
and HR is 137 bpm, O2 sat was 95% on room air. On physical examination shows an irregularly
irregular rhythm. Auscultation reveals no crackles. Which of the following is the best next step
in management of this patient?
a) Adenosine
O
b) Metoprolol
C) Amiodarone
d) Heparin
e) Synchronized cardioversion
76.A 65 year old woman has a longstanding history of primary hypothyroidism with poor
compliance to treatment. She presents to ER with decreased level of consciousness and you are
concerned of her having myxedema coma. Which of the following abnormality will support
your working diagnosis?
a) Hypernatremia
x
O
b) Hypercapnia
c) Hypercalcemia
d) Hyperthermia
x
77.50 year old male presents with chest pain that occurred 2 hours ago, ECG shows ST elevation
V1-V4.
Which of the following are contraindications to thrombolysis?
A. Creatinine> 2.8
B. Systolic BP> 200
C. Absence of chest pain at present time
D. Gastrectomy for GI bleed 6 months ago
E. No contraindication
78. A 54 year old man has a 3 hour history of chest pain and dyspnea. He is 1 - week post
colectomy for colon cancer. On examination, Temp 37.5ºC, BP 110/60 mmHg, HR 130 / min,
and RR 24 / min. Oxygen saturation is 88% on room air and 94% on oxygen, 4 L / min. Cardiac
examination shows tachycardia. Breath sounds are normal. Serum creatinine concentration is 3
mg / dl (normal 0.84 1.2 mg / dl) and his complete blood count and coagulation profile is normal.
CT pulmonary angiography shows left sided pulmonary embolism. Which of the following is
the most appropriate immediate treatment?
A. Unfractionated heparin
B. Rivaroxiban
C. Enoxaparin
D. Fondaparinux
E. Dabigatran
79. A 20 year old male, with a medical history significant for active intravenous drug user
presented to emergency room complaining of persistent fever. Pertinent positive on physical
examination are temperature of 39.3 ° C, multiple pink, painful nodules on the palmar surface of
both hands and grade 3 pansystolic murmur on fourth left parasternal area. Three sets of blood
culture are sent What is the most likely cause of persistent fever?
A. Infective endocarditis
B. Lung abscess
C. Acute rheumatic fever
D. Drug induced fever
E. Allergic reaction
80. You were called to manage a 35-year male motor vehicle accident victim who has sustained
multiple leg fractures in addition to splenic and liver lacerations few days ago
The patient developed sudden shortness of breath and central chest pain worsening with deep
breathing No associated cough, wheeze or fever. His legs had been in casts from mid tibia till
above knee. BP 90/60) HR 130 regular, Temp 37, RR32, Oxygen Saturation 88% on room air.
Chest examination revealed central trachea, symmetrical air entry without added sounds. CVS
exam showed elevated JVP with loud pulmonary component of second sound. Leg exams
showed diameter difference at mid thigh showing larger left leg. Laboratory work up showed
drop in hemoglobin from 10 to 8 and stable WBC of 12000 in addition to normal cardiac
enzymes and chest X ray. The most appropriate next step is:
a) D dimer testing
b) Doppler US both legs
c) Echocardiogram
a
d) Start anticoagulant empirically
e) Transfuse packed red blood cells